A 55-year-old female presented a with few years' history of pruritic plaques on her shins and wrists

Article Type
Changed
Fri, 12/15/2023 - 16:18

Hypertrophic lichen planus (HLP), also known as LP verrucosus, is a form of lichen planus where lesions are lichenified and thicker than the typical flat, purple, polygonal lesions found in lichen planus. Lesions may have a covering of scale. HLP commonly affects middle aged men and women. Lesions are most commonly located bilaterally on the shins and ankles and can be painful or pruritic. The differential diagnosis for the condition includes lichen simplex chronicus, connective tissue disease, and other skin disorders that cause hyperkeratosis. This wide differential makes histopathological analysis a useful tool in confirming the diagnosis of HLP.

A definitive diagnosis can be made via skin biopsy. Histopathology reveals hyperkeratosis, acanthosis, and a band-like lymphocytic infiltrate in the dermis. An eosinophilic infiltrate may be present. Other common features include saw tooth rete ridges and Civatte bodies, which are apoptotic keratinocytes. The lymphocytic infiltrate may indicate an autoimmune etiology in which the body’s immune system erroneously attacks itself. However, the exact cause is not known and genetic and environmental factors may play a role.

The treatment of HLP includes symptomatic management and control of inflammation. Topical steroids can be prescribed to manage the inflammation and associated pruritus, and emollient creams and moisturizers are helpful in controlling the dryness. Oral steroids, immunosuppressant medications, or retinoids may be necessary in more severe cases. In addition, psoralen plus ultraviolet A (PUVA) light therapy has been found to be beneficial in some cases. Squamous cell carcinoma may arise in lesions.

BiluMartin_Donna_FLORIDA_web.jpg
Dr. Donna Bilu Martin


This case and photo were submitted by Lucas Shapiro, BS, of Nova Southeastern University College of Osteopathic Medicine, Fort Lauderdale, Florida, and Donna Bilu Martin, MD; Premier Dermatology, MD, Aventura, Florida. The column was edited by Dr. Bilu Martin.
 

Dr. Bilu Martin is a board-certified dermatologist in private practice at Premier Dermatology, MD, in Aventura, Fla. More diagnostic cases are available at mdedge.com/dermatology. To submit a case for possible publication, send an email to dermnews@mdedge.com.

References

Arnold DL, Krishnamurthy K. Lichen Planus. [Updated 2023 Jun 1]. In: StatPearls [Internet]. Treasure Island (FL): StatPearls Publishing; 2023 Jan-. Available from: https://www.ncbi.nlm.nih.gov/books/NBK526126/

Jaime TJ et al. An Bras Dermatol. 2011 Jul-Aug;86(4 Suppl 1):S96-9.

Mirchandani S et al. Med Pharm Rep. 2020 Apr;93(2):210-2. .

Whittington CP et al. Arch Pathol Lab Med. 2023 Jun 19. doi: 10.5858/arpa.2022-0515-RA.

Publications
Topics
Sections

Hypertrophic lichen planus (HLP), also known as LP verrucosus, is a form of lichen planus where lesions are lichenified and thicker than the typical flat, purple, polygonal lesions found in lichen planus. Lesions may have a covering of scale. HLP commonly affects middle aged men and women. Lesions are most commonly located bilaterally on the shins and ankles and can be painful or pruritic. The differential diagnosis for the condition includes lichen simplex chronicus, connective tissue disease, and other skin disorders that cause hyperkeratosis. This wide differential makes histopathological analysis a useful tool in confirming the diagnosis of HLP.

A definitive diagnosis can be made via skin biopsy. Histopathology reveals hyperkeratosis, acanthosis, and a band-like lymphocytic infiltrate in the dermis. An eosinophilic infiltrate may be present. Other common features include saw tooth rete ridges and Civatte bodies, which are apoptotic keratinocytes. The lymphocytic infiltrate may indicate an autoimmune etiology in which the body’s immune system erroneously attacks itself. However, the exact cause is not known and genetic and environmental factors may play a role.

The treatment of HLP includes symptomatic management and control of inflammation. Topical steroids can be prescribed to manage the inflammation and associated pruritus, and emollient creams and moisturizers are helpful in controlling the dryness. Oral steroids, immunosuppressant medications, or retinoids may be necessary in more severe cases. In addition, psoralen plus ultraviolet A (PUVA) light therapy has been found to be beneficial in some cases. Squamous cell carcinoma may arise in lesions.

BiluMartin_Donna_FLORIDA_web.jpg
Dr. Donna Bilu Martin


This case and photo were submitted by Lucas Shapiro, BS, of Nova Southeastern University College of Osteopathic Medicine, Fort Lauderdale, Florida, and Donna Bilu Martin, MD; Premier Dermatology, MD, Aventura, Florida. The column was edited by Dr. Bilu Martin.
 

Dr. Bilu Martin is a board-certified dermatologist in private practice at Premier Dermatology, MD, in Aventura, Fla. More diagnostic cases are available at mdedge.com/dermatology. To submit a case for possible publication, send an email to dermnews@mdedge.com.

References

Arnold DL, Krishnamurthy K. Lichen Planus. [Updated 2023 Jun 1]. In: StatPearls [Internet]. Treasure Island (FL): StatPearls Publishing; 2023 Jan-. Available from: https://www.ncbi.nlm.nih.gov/books/NBK526126/

Jaime TJ et al. An Bras Dermatol. 2011 Jul-Aug;86(4 Suppl 1):S96-9.

Mirchandani S et al. Med Pharm Rep. 2020 Apr;93(2):210-2. .

Whittington CP et al. Arch Pathol Lab Med. 2023 Jun 19. doi: 10.5858/arpa.2022-0515-RA.

Hypertrophic lichen planus (HLP), also known as LP verrucosus, is a form of lichen planus where lesions are lichenified and thicker than the typical flat, purple, polygonal lesions found in lichen planus. Lesions may have a covering of scale. HLP commonly affects middle aged men and women. Lesions are most commonly located bilaterally on the shins and ankles and can be painful or pruritic. The differential diagnosis for the condition includes lichen simplex chronicus, connective tissue disease, and other skin disorders that cause hyperkeratosis. This wide differential makes histopathological analysis a useful tool in confirming the diagnosis of HLP.

A definitive diagnosis can be made via skin biopsy. Histopathology reveals hyperkeratosis, acanthosis, and a band-like lymphocytic infiltrate in the dermis. An eosinophilic infiltrate may be present. Other common features include saw tooth rete ridges and Civatte bodies, which are apoptotic keratinocytes. The lymphocytic infiltrate may indicate an autoimmune etiology in which the body’s immune system erroneously attacks itself. However, the exact cause is not known and genetic and environmental factors may play a role.

The treatment of HLP includes symptomatic management and control of inflammation. Topical steroids can be prescribed to manage the inflammation and associated pruritus, and emollient creams and moisturizers are helpful in controlling the dryness. Oral steroids, immunosuppressant medications, or retinoids may be necessary in more severe cases. In addition, psoralen plus ultraviolet A (PUVA) light therapy has been found to be beneficial in some cases. Squamous cell carcinoma may arise in lesions.

BiluMartin_Donna_FLORIDA_web.jpg
Dr. Donna Bilu Martin


This case and photo were submitted by Lucas Shapiro, BS, of Nova Southeastern University College of Osteopathic Medicine, Fort Lauderdale, Florida, and Donna Bilu Martin, MD; Premier Dermatology, MD, Aventura, Florida. The column was edited by Dr. Bilu Martin.
 

Dr. Bilu Martin is a board-certified dermatologist in private practice at Premier Dermatology, MD, in Aventura, Fla. More diagnostic cases are available at mdedge.com/dermatology. To submit a case for possible publication, send an email to dermnews@mdedge.com.

References

Arnold DL, Krishnamurthy K. Lichen Planus. [Updated 2023 Jun 1]. In: StatPearls [Internet]. Treasure Island (FL): StatPearls Publishing; 2023 Jan-. Available from: https://www.ncbi.nlm.nih.gov/books/NBK526126/

Jaime TJ et al. An Bras Dermatol. 2011 Jul-Aug;86(4 Suppl 1):S96-9.

Mirchandani S et al. Med Pharm Rep. 2020 Apr;93(2):210-2. .

Whittington CP et al. Arch Pathol Lab Med. 2023 Jun 19. doi: 10.5858/arpa.2022-0515-RA.

Publications
Publications
Topics
Article Type
Sections
Teambase XML
<?xml version="1.0" encoding="UTF-8"?>
<!--$RCSfile: InCopy_agile.xsl,v $ $Revision: 1.35 $-->
<!--$RCSfile: drupal.xsl,v $ $Revision: 1.7 $-->
<root generator="drupal.xsl" gversion="1.7"> <header> <fileName>166301</fileName> <TBEID>0C04DB42.SIG</TBEID> <TBUniqueIdentifier>MD_0C04DB42</TBUniqueIdentifier> <newsOrJournal>News</newsOrJournal> <publisherName>Frontline Medical Communications</publisherName> <storyname/> <articleType>353</articleType> <TBLocation>QC Done-All Pubs</TBLocation> <QCDate>20231215T145928</QCDate> <firstPublished>20231215T160752</firstPublished> <LastPublished>20231215T160752</LastPublished> <pubStatus qcode="stat:"/> <embargoDate/> <killDate/> <CMSDate>20231215T160752</CMSDate> <articleSource/> <facebookInfo/> <meetingNumber/> <byline>Shapiro and Bilu Martin</byline> <bylineText>LUCAS SHAPIRO, BS, AND DONNA BILU MARTIN, MD</bylineText> <bylineFull>LUCAS SHAPIRO, BS, AND DONNA BILU MARTIN, MD</bylineFull> <bylineTitleText/> <USOrGlobal/> <wireDocType/> <newsDocType/> <journalDocType/> <linkLabel/> <pageRange/> <citation/> <quizID/> <indexIssueDate/> <itemClass qcode="ninat:text"/> <provider qcode="provider:imng"> <name>IMNG Medical Media</name> <rightsInfo> <copyrightHolder> <name>Frontline Medical News</name> </copyrightHolder> <copyrightNotice>Copyright (c) 2015 Frontline Medical News, a Frontline Medical Communications Inc. company. All rights reserved. This material may not be published, broadcast, copied, or otherwise reproduced or distributed without the prior written permission of Frontline Medical Communications Inc.</copyrightNotice> </rightsInfo> </provider> <abstract/> <metaDescription>Hypertrophic lichen planus (HLP), also known as LP verrucosus, is a form of lichen planus where lesions are lichenified and thicker than the typical flat, purpl</metaDescription> <articlePDF/> <teaserImage>299706</teaserImage> <title>Hypertrophic lichen planus</title> <deck/> <disclaimer/> <AuthorList/> <articleURL/> <doi/> <pubMedID/> <publishXMLStatus/> <publishXMLVersion>1</publishXMLVersion> <useEISSN>0</useEISSN> <urgency/> <pubPubdateYear/> <pubPubdateMonth/> <pubPubdateDay/> <pubVolume/> <pubNumber/> <wireChannels/> <primaryCMSID/> <CMSIDs/> <keywords/> <seeAlsos/> <publications_g> <publicationData> <publicationCode>skin</publicationCode> <pubIssueName/> <pubArticleType/> <pubTopics/> <pubCategories/> <pubSections/> </publicationData> <publicationData> <publicationCode>im</publicationCode> <pubIssueName/> <pubArticleType/> <pubTopics/> <pubCategories/> <pubSections/> </publicationData> <publicationData> <publicationCode>fp</publicationCode> <pubIssueName/> <pubArticleType/> <pubTopics/> <pubCategories/> <pubSections/> </publicationData> </publications_g> <publications> <term canonical="true">13</term> <term>21</term> <term>15</term> </publications> <sections> <term>52</term> <term canonical="true">87</term> </sections> <topics> <term canonical="true">39212</term> <term>203</term> </topics> <links> <link> <itemClass qcode="ninat:picture"/> <altRep contenttype="image/jpeg">images/2401252b.jpg</altRep> <description role="drol:caption"/> <description role="drol:credit">Lucas Shapiro and Donna Bilu Martin</description> </link> <link> <itemClass qcode="ninat:picture"/> <altRep contenttype="image/jpeg">images/2400f1a6.jpg</altRep> <description role="drol:caption">Dr. Donna Bilu Martin</description> <description role="drol:credit"/> </link> </links> </header> <itemSet> <newsItem> <itemMeta> <itemRole>Main</itemRole> <itemClass>text</itemClass> <title>Hypertrophic lichen planus</title> <deck/> </itemMeta> <itemContent> <p><br/><br/><span class="tag metaDescription">Hypertrophic lichen planus (HLP), also known as LP verrucosus, is a form of lichen planus where lesions are lichenified and thicker than the typical flat, purple, polygonal lesions found in lichen planus</span>. Lesions may have a covering of scale. HLP commonly affects middle aged men and women. Lesions are most commonly located bilaterally on the shins and ankles and can be painful or pruritic. The differential diagnosis for the condition includes lichen simplex chronicus, connective tissue disease, and other skin disorders that cause hyperkeratosis. This wide differential makes histopathological analysis a useful tool in confirming the diagnosis of HLP.<br/><br/>A definitive diagnosis can be made via skin biopsy. Histopathology reveals hyperkeratosis, acanthosis, and a band-like lymphocytic infiltrate in the dermis. An eosinophilic infiltrate may be present. Other common features include saw tooth rete ridges and Civatte bodies, which are apoptotic keratinocytes. The lymphocytic infiltrate may indicate an autoimmune etiology in which the body’s immune system erroneously attacks itself. However, the exact cause is not known and genetic and environmental factors may play a role. <br/><br/>[[{"fid":"299706","view_mode":"medstat_image_flush_right","fields":{"format":"medstat_image_flush_right","field_file_image_alt_text[und][0][value]":"","field_file_image_credit[und][0][value]":"Lucas Shapiro and Donna Bilu Martin","field_file_image_caption[und][0][value]":""},"type":"media","attributes":{"class":"media-element file-medstat_image_flush_right"}}]]The treatment of HLP includes symptomatic management and control of inflammation. Topical steroids can be prescribed to manage the inflammation and associated pruritus, and emollient creams and moisturizers are helpful in controlling the dryness. Oral steroids, immunosuppressant medications, or retinoids may be necessary in more severe cases. In addition, psoralen plus ultraviolet A (PUVA) light therapy has been found to be beneficial in some cases. Squamous cell carcinoma may arise in lesions.[[{"fid":"271802","view_mode":"medstat_image_flush_right","fields":{"format":"medstat_image_flush_right","field_file_image_alt_text[und][0][value]":"Dr. Donna Bilu Martin, Premier Dermatology, MD, Aventura, Fla.","field_file_image_credit[und][0][value]":"","field_file_image_caption[und][0][value]":"Dr. Donna Bilu Martin"},"type":"media","attributes":{"class":"media-element file-medstat_image_flush_right"}}]]<br/><br/>This case and photo were submitted by Lucas Shapiro, BS, of Nova Southeastern University College of Osteopathic Medicine, Fort Lauderdale, Florida, and Donna Bilu Martin, MD; Premier Dermatology, MD, Aventura, Florida. The column was edited by Dr. Bilu Martin.<br/><br/></p> <p> <em>Dr. Bilu Martin is a board-certified dermatologist in private practice at Premier Dermatology, MD, in Aventura, Fla. More diagnostic cases are available at <span class="Hyperlink"><a href="http://mdedge.com/dermatology">mdedge.com/dermatology</a></span>. To submit a case for possible publication, send an email to <span class="Hyperlink"><a href="mailto:dermnews%40mdedge.com?subject=">dermnews@mdedge.com</a></span>.</em> </p> <h2>References</h2> <p>Arnold DL, Krishnamurthy K. <span class="Hyperlink"><a href="https://www.ncbi.nlm.nih.gov/books/NBK526126/">Lichen Planus</a></span>. [Updated 2023 Jun 1]. In: StatPearls [Internet]. Treasure Island (FL): StatPearls Publishing; 2023 Jan-. Available from: https://www.ncbi.nlm.nih.gov/books/NBK526126/<br/><br/>Jaime TJ et al. <span class="Hyperlink"><a href="https://www.scielo.br/j/abd/a/76S7p5mysv3cWVgY5t6GZKh/abstract/?lang=en">An Bras Dermatol. 2011 Jul-Aug;86(4 Suppl 1):S96-9</a></span>.<br/><br/>Mirchandani S et al. <span class="Hyperlink"><a href="https://pubmed.ncbi.nlm.nih.gov/32478329/">Med Pharm Rep. 2020 Apr;93(2):210-2</a></span>. .<br/><br/>Whittington CP et al. <span class="Hyperlink"><a href="https://pubmed.ncbi.nlm.nih.gov/37327190/">Arch Pathol Lab Med. 2023 Jun 19. doi: 10.5858/arpa.2022-0515-RA</a></span>.</p> </itemContent> </newsItem> <newsItem> <itemMeta> <itemRole>teaser</itemRole> <itemClass>text</itemClass> <title/> <deck/> </itemMeta> <itemContent> <p>Histopathology reveals hyperkeratosis, acanthosis, and a band-like lymphocytic infiltrate in the dermis.</p> </itemContent> </newsItem> </itemSet></root>
Questionnaire Body

166301_photo_web.jpg
A 55-year-old female with no significant medical history presented with a few years history of thick, pruritic, erythematous plaques on her shins and wrists that come and go. A biopsy was previously performed.

Disallow All Ads
Content Gating
No Gating (article Unlocked/Free)
Alternative CME
Disqus Comments
Default
Use ProPublica
Hide sidebar & use full width
render the right sidebar.
Conference Recap Checkbox
Not Conference Recap
Clinical Edge
Display the Slideshow in this Article
Medscape Article
Display survey writer
Reuters content
Disable Inline Native ads
WebMD Article

An 88-year-old Black woman presented with 3 months duration of asymptomatic, violaceous patches on the left breast

Article Type
Changed
Thu, 11/09/2023 - 13:14

Angiosarcomas are uncommon, high-grade malignant tumors of endothelial cell origin that can arise via the lymphatics or vasculature. They typically occur spontaneously; however, there have been cases reported of benign vascular transformation. These tumors are more commonly found in elderly men on the head and neck in sun-damaged skin. Radiation-induced angiosarcoma occurs in an estimated 0.05%-0.3% of patients with breast cancer who underwent surgery and radiation therapy. This is a late complication, typically occurring about 5-10 years after radiation. Stewart-Treves syndrome, chronic lymphedema occurring after breast cancer treatment with axillary node dissection, increases the risk of angiosarcoma. As a vascular tumor, angiosarcoma spreads hematogenously and carries a poor prognosis if not caught early. Differential diagnoses include other vascular tumors such as retiform hemangioendothelioma. In this specific patient, the differential diagnosis includes Paget’s disease, chronic radiation skin changes, and eczema.

BiluMartin_Donna_FLORIDA_web.jpg
Dr. Donna Bilu Martin

Histopathologically, angiosarcomas exhibit abnormal, pleomorphic, malignant endothelial cells. As the tumor progresses, the cell architecture becomes more distorted and cells form layers with papillary projections into the vascular lumen. Malignant cells may stain positive for CD31, CD34, the oncogene ERG and the proto-oncogene FLI-1. Histology in this patient revealed radiation changes in the dermis, as well as few vascular channels lined by large endothelial cells with marked nuclear atypia, in the form of large nucleoli and variably coarse chromatin. The cells were positive for MYC.

Treatment of angiosarcoma involves a multidisciplinary approach. Resection with wide margins is generally the treatment of choice. However, recurrence is relatively common, which may be a result of microsatellite deposits of the tumor. Perioperative radiation is recommended, and adjuvant chemotherapy often is recommended for metastatic disease. Specifically, paclitaxel has been found to promote survival in some cases of cutaneous angiosarcoma. Metastatic disease may be treated with cytotoxic drugs such as anthracyclines and taxanes. Additionally, targeted therapy including anti-VEGF drugs and tyrosine kinase inhibitors have been tested.

The case and photo were submitted by Mr. Shapiro of Nova Southeastern University College of Osteopathic Medicine, Fort Lauderdale, Fla., and Dr. Bilu Martin. The column was edited by Dr. Bilu Martin.

Dr. Bilu Martin is a board-certified dermatologist in private practice at Premier Dermatology, MD, in Aventura, Fla. More diagnostic cases are available at mdedge.com/dermatology. To submit a case for possible publication, send an email to dermnews@mdedge.com.

References

Cohen-Hallaleh RB et al. Clin Sarcoma Res. 2017 Aug 7:7:15.

Cozzi S et al. Rep Pract Oncol Radiother. 2021 Sep 30;26(5):827-32.

Spiker AM, Mangla A, Ramsey ML. Angiosarcoma. [Updated 2023 Jul 17]. In: StatPearls [Internet]. Treasure Island, Fla.: StatPearls Publishing; 2023 Jan-. Available from: www.ncbi.nlm.nih.gov/books/NBK441983/

Publications
Topics
Sections

Angiosarcomas are uncommon, high-grade malignant tumors of endothelial cell origin that can arise via the lymphatics or vasculature. They typically occur spontaneously; however, there have been cases reported of benign vascular transformation. These tumors are more commonly found in elderly men on the head and neck in sun-damaged skin. Radiation-induced angiosarcoma occurs in an estimated 0.05%-0.3% of patients with breast cancer who underwent surgery and radiation therapy. This is a late complication, typically occurring about 5-10 years after radiation. Stewart-Treves syndrome, chronic lymphedema occurring after breast cancer treatment with axillary node dissection, increases the risk of angiosarcoma. As a vascular tumor, angiosarcoma spreads hematogenously and carries a poor prognosis if not caught early. Differential diagnoses include other vascular tumors such as retiform hemangioendothelioma. In this specific patient, the differential diagnosis includes Paget’s disease, chronic radiation skin changes, and eczema.

BiluMartin_Donna_FLORIDA_web.jpg
Dr. Donna Bilu Martin

Histopathologically, angiosarcomas exhibit abnormal, pleomorphic, malignant endothelial cells. As the tumor progresses, the cell architecture becomes more distorted and cells form layers with papillary projections into the vascular lumen. Malignant cells may stain positive for CD31, CD34, the oncogene ERG and the proto-oncogene FLI-1. Histology in this patient revealed radiation changes in the dermis, as well as few vascular channels lined by large endothelial cells with marked nuclear atypia, in the form of large nucleoli and variably coarse chromatin. The cells were positive for MYC.

Treatment of angiosarcoma involves a multidisciplinary approach. Resection with wide margins is generally the treatment of choice. However, recurrence is relatively common, which may be a result of microsatellite deposits of the tumor. Perioperative radiation is recommended, and adjuvant chemotherapy often is recommended for metastatic disease. Specifically, paclitaxel has been found to promote survival in some cases of cutaneous angiosarcoma. Metastatic disease may be treated with cytotoxic drugs such as anthracyclines and taxanes. Additionally, targeted therapy including anti-VEGF drugs and tyrosine kinase inhibitors have been tested.

The case and photo were submitted by Mr. Shapiro of Nova Southeastern University College of Osteopathic Medicine, Fort Lauderdale, Fla., and Dr. Bilu Martin. The column was edited by Dr. Bilu Martin.

Dr. Bilu Martin is a board-certified dermatologist in private practice at Premier Dermatology, MD, in Aventura, Fla. More diagnostic cases are available at mdedge.com/dermatology. To submit a case for possible publication, send an email to dermnews@mdedge.com.

References

Cohen-Hallaleh RB et al. Clin Sarcoma Res. 2017 Aug 7:7:15.

Cozzi S et al. Rep Pract Oncol Radiother. 2021 Sep 30;26(5):827-32.

Spiker AM, Mangla A, Ramsey ML. Angiosarcoma. [Updated 2023 Jul 17]. In: StatPearls [Internet]. Treasure Island, Fla.: StatPearls Publishing; 2023 Jan-. Available from: www.ncbi.nlm.nih.gov/books/NBK441983/

Angiosarcomas are uncommon, high-grade malignant tumors of endothelial cell origin that can arise via the lymphatics or vasculature. They typically occur spontaneously; however, there have been cases reported of benign vascular transformation. These tumors are more commonly found in elderly men on the head and neck in sun-damaged skin. Radiation-induced angiosarcoma occurs in an estimated 0.05%-0.3% of patients with breast cancer who underwent surgery and radiation therapy. This is a late complication, typically occurring about 5-10 years after radiation. Stewart-Treves syndrome, chronic lymphedema occurring after breast cancer treatment with axillary node dissection, increases the risk of angiosarcoma. As a vascular tumor, angiosarcoma spreads hematogenously and carries a poor prognosis if not caught early. Differential diagnoses include other vascular tumors such as retiform hemangioendothelioma. In this specific patient, the differential diagnosis includes Paget’s disease, chronic radiation skin changes, and eczema.

BiluMartin_Donna_FLORIDA_web.jpg
Dr. Donna Bilu Martin

Histopathologically, angiosarcomas exhibit abnormal, pleomorphic, malignant endothelial cells. As the tumor progresses, the cell architecture becomes more distorted and cells form layers with papillary projections into the vascular lumen. Malignant cells may stain positive for CD31, CD34, the oncogene ERG and the proto-oncogene FLI-1. Histology in this patient revealed radiation changes in the dermis, as well as few vascular channels lined by large endothelial cells with marked nuclear atypia, in the form of large nucleoli and variably coarse chromatin. The cells were positive for MYC.

Treatment of angiosarcoma involves a multidisciplinary approach. Resection with wide margins is generally the treatment of choice. However, recurrence is relatively common, which may be a result of microsatellite deposits of the tumor. Perioperative radiation is recommended, and adjuvant chemotherapy often is recommended for metastatic disease. Specifically, paclitaxel has been found to promote survival in some cases of cutaneous angiosarcoma. Metastatic disease may be treated with cytotoxic drugs such as anthracyclines and taxanes. Additionally, targeted therapy including anti-VEGF drugs and tyrosine kinase inhibitors have been tested.

The case and photo were submitted by Mr. Shapiro of Nova Southeastern University College of Osteopathic Medicine, Fort Lauderdale, Fla., and Dr. Bilu Martin. The column was edited by Dr. Bilu Martin.

Dr. Bilu Martin is a board-certified dermatologist in private practice at Premier Dermatology, MD, in Aventura, Fla. More diagnostic cases are available at mdedge.com/dermatology. To submit a case for possible publication, send an email to dermnews@mdedge.com.

References

Cohen-Hallaleh RB et al. Clin Sarcoma Res. 2017 Aug 7:7:15.

Cozzi S et al. Rep Pract Oncol Radiother. 2021 Sep 30;26(5):827-32.

Spiker AM, Mangla A, Ramsey ML. Angiosarcoma. [Updated 2023 Jul 17]. In: StatPearls [Internet]. Treasure Island, Fla.: StatPearls Publishing; 2023 Jan-. Available from: www.ncbi.nlm.nih.gov/books/NBK441983/

Publications
Publications
Topics
Article Type
Sections
Teambase XML
<?xml version="1.0" encoding="UTF-8"?>
<!--$RCSfile: InCopy_agile.xsl,v $ $Revision: 1.35 $-->
<!--$RCSfile: drupal.xsl,v $ $Revision: 1.7 $-->
<root generator="drupal.xsl" gversion="1.7"> <header> <fileName>165858</fileName> <TBEID>0C04D1E6.SIG</TBEID> <TBUniqueIdentifier>MD_0C04D1E6</TBUniqueIdentifier> <newsOrJournal>News</newsOrJournal> <publisherName>Frontline Medical Communications</publisherName> <storyname>December Make the Dx</storyname> <articleType>353</articleType> <TBLocation>QC Done-All Pubs</TBLocation> <QCDate>20231109T125556</QCDate> <firstPublished>20231109T130731</firstPublished> <LastPublished>20231109T130731</LastPublished> <pubStatus qcode="stat:"/> <embargoDate/> <killDate/> <CMSDate>20231109T130731</CMSDate> <articleSource/> <facebookInfo/> <meetingNumber/> <byline>Shapiro and Bilu Martin</byline> <bylineText>LUCAS SHAPIRO, BS, AND DONNA BILU MARTIN, MD</bylineText> <bylineFull>LUCAS SHAPIRO, BS, AND DONNA BILU MARTIN, MD</bylineFull> <bylineTitleText/> <USOrGlobal/> <wireDocType/> <newsDocType>Column</newsDocType> <journalDocType/> <linkLabel/> <pageRange/> <citation/> <quizID/> <indexIssueDate/> <itemClass qcode="ninat:text"/> <provider qcode="provider:imng"> <name>IMNG Medical Media</name> <rightsInfo> <copyrightHolder> <name>Frontline Medical News</name> </copyrightHolder> <copyrightNotice>Copyright (c) 2015 Frontline Medical News, a Frontline Medical Communications Inc. company. All rights reserved. This material may not be published, broadcast, copied, or otherwise reproduced or distributed without the prior written permission of Frontline Medical Communications Inc.</copyrightNotice> </rightsInfo> </provider> <abstract/> <metaDescription>Radiation-induced angiosarcoma occurs in an estimated 0.05%-0.3% of patients with breast cancer who underwent surgery and radiation therapy</metaDescription> <articlePDF/> <teaserImage>271802</teaserImage> <teaser>Angiosarcomas are uncommon, high-grade malignant tumors of endothelial cell origin that can arise via the lymphatics or vasculature.</teaser> <title>Radiation-induced angiosarcoma</title> <deck/> <disclaimer/> <AuthorList/> <articleURL/> <doi/> <pubMedID/> <publishXMLStatus/> <publishXMLVersion>1</publishXMLVersion> <useEISSN>0</useEISSN> <urgency/> <pubPubdateYear/> <pubPubdateMonth/> <pubPubdateDay/> <pubVolume/> <pubNumber/> <wireChannels/> <primaryCMSID/> <CMSIDs/> <keywords/> <seeAlsos/> <publications_g> <publicationData> <publicationCode>skin</publicationCode> <pubIssueName/> <pubArticleType/> <pubTopics/> <pubCategories/> <pubSections/> </publicationData> <publicationData> <publicationCode>im</publicationCode> <pubIssueName/> <pubArticleType/> <pubTopics/> <pubCategories/> <pubSections/> </publicationData> <publicationData> <publicationCode>fp</publicationCode> <pubIssueName/> <pubArticleType/> <pubTopics/> <pubCategories/> <pubSections/> </publicationData> <publicationData> <publicationCode>oncr</publicationCode> <pubIssueName/> <pubArticleType/> <pubTopics/> <pubCategories/> <pubSections/> </publicationData> </publications_g> <publications> <term canonical="true">13</term> <term>21</term> <term>15</term> <term>31</term> </publications> <sections> <term>52</term> <term canonical="true">87</term> </sections> <topics> <term canonical="true">39212</term> <term>203</term> <term>192</term> <term>263</term> <term>292</term> <term>204</term> </topics> <links> <link> <itemClass qcode="ninat:picture"/> <altRep contenttype="image/jpeg">images/2400f1a6.jpg</altRep> <description role="drol:caption">Dr. Donna Bilu Martin</description> <description role="drol:credit"/> </link> </links> </header> <itemSet> <newsItem> <itemMeta> <itemRole>Main</itemRole> <itemClass>text</itemClass> <title>Radiation-induced angiosarcoma</title> <deck/> </itemMeta> <itemContent> <p>Angiosarcomas are uncommon, high-grade malignant tumors of endothelial cell origin that can arise via the lymphatics or vasculature. They typically occur spontaneously; however, there have been cases reported of benign vascular transformation. These tumors are more commonly found in elderly men on the head and neck in sun-damaged skin. <span class="tag metaDescription">Radiation-induced angiosarcoma occurs in an estimated 0.05%-0.3% of patients with breast cancer who underwent surgery and radiation therapy</span>. This is a late complication, typically occurring about 5-10 years after radiation. Stewart-Treves syndrome, chronic lymphedema occurring after breast cancer treatment with axillary node dissection, increases the risk of angiosarcoma. As a vascular tumor, angiosarcoma spreads hematogenously and carries a poor prognosis if not caught early. Differential diagnoses include other vascular tumors such as retiform hemangioendothelioma. In this specific patient, the differential diagnosis includes Paget’s disease, chronic radiation skin changes, and eczema. </p> <p>[[{"fid":"271802","view_mode":"medstat_image_flush_right","fields":{"format":"medstat_image_flush_right","field_file_image_alt_text[und][0][value]":"Dr. Donna Bilu Martin, Premier Dermatology, MD, Aventura, Fla.","field_file_image_credit[und][0][value]":"","field_file_image_caption[und][0][value]":"Dr. Donna Bilu Martin"},"type":"media","attributes":{"class":"media-element file-medstat_image_flush_right"}}]]Histopathologically, angiosarcomas exhibit abnormal, pleomorphic, malignant endothelial cells. As the tumor progresses, the cell architecture becomes more distorted and cells form layers with papillary projections into the vascular lumen. Malignant cells may stain positive for CD31, CD34, the oncogene ERG and the proto-oncogene <span class="Hyperlink"><a href="https://medlineplus.gov/genetics/gene/fli1/">FLI-1</a></span>. Histology in this patient revealed radiation changes in the dermis, as well as few vascular channels lined by large endothelial cells with marked nuclear atypia, in the form of large nucleoli and variably coarse chromatin. The cells were positive for MYC. <br/><br/>Treatment of angiosarcoma involves a multidisciplinary approach. Resection with wide margins is generally the treatment of choice. However, recurrence is relatively common, which may be a result of microsatellite deposits of the tumor. Perioperative radiation is recommended, and adjuvant chemotherapy often is recommended for metastatic disease. Specifically, paclitaxel has been found to promote survival in some cases of cutaneous angiosarcoma. Metastatic disease may be treated with cytotoxic drugs such as anthracyclines and taxanes. Additionally, targeted therapy including anti-VEGF drugs and tyrosine kinase inhibitors have been tested.</p> <p>The case and photo were submitted by Mr. Shapiro of Nova Southeastern University College of Osteopathic Medicine, Fort Lauderdale, Fla., and Dr. Bilu Martin. The column was edited by Dr. Bilu Martin.<span class="end"/></p> <p> <em>Dr. Bilu Martin is a board-certified dermatologist in private practice at Premier Dermatology, MD, in Aventura, Fla. More diagnostic cases are available at <span class="Hyperlink"><a href="http://mdedge.com/dermatology">mdedge.com/dermatology</a></span>. To submit a case for possible publication, send an email to <span class="Hyperlink"><a href="mailto:dermnews%40mdedge.com?subject=">dermnews@mdedge.com</a></span>.</em> </p> <h2>References</h2> <p>Cohen-Hallaleh RB et al. <span class="Hyperlink"><a href="https://www.ncbi.nlm.nih.gov/pmc/articles/PMC5547463/">Clin Sarcoma Res. 2017 Aug 7:7:15</a></span>.<br/><br/>Cozzi S et al. <span class="Hyperlink"><a href="https://www.ncbi.nlm.nih.gov/pmc/articles/PMC8575351/">Rep Pract Oncol Radiother. 2021 Sep 30;26(5):827-32</a></span>.<br/><br/>Spiker AM, Mangla A, Ramsey ML. <span class="Hyperlink"><a href="https://www.ncbi.nlm.nih.gov/books/NBK441983/">Angiosarcoma</a></span>. [Updated 2023 Jul 17]. In: StatPearls [Internet]. Treasure Island, Fla.: StatPearls Publishing; 2023 Jan-. Available from: <span class="Hyperlink"><a href="https://www.ncbi.nlm.nih.gov/books/NBK441983/">www.ncbi.nlm.nih.gov/books/NBK441983/</a></span> <span class="Hyperlink"/></p> </itemContent> </newsItem> <newsItem> <itemMeta> <itemRole>teaser</itemRole> <itemClass>text</itemClass> <title/> <deck/> </itemMeta> <itemContent> </itemContent> </newsItem> </itemSet></root>
Questionnaire Body

165858_angiosarcoma_web.jpg
An 88-year-old woman Black woman presented with 3 months duration of asymptomatic, violaceous patches on the left breast. The patient's history is significant for breast cancer treated with surgery and radiation 6 years ago.

Disallow All Ads
Content Gating
No Gating (article Unlocked/Free)
Alternative CME
Disqus Comments
Default
Use ProPublica
Hide sidebar & use full width
render the right sidebar.
Conference Recap Checkbox
Not Conference Recap
Clinical Edge
Display the Slideshow in this Article
Medscape Article
Display survey writer
Reuters content
Disable Inline Native ads
WebMD Article

A 45-year-old White woman with no significant medical history presented with a 1-month history of lesions on the nose and right cheek

Article Type
Changed
Mon, 08/07/2023 - 12:13

Cultures for bacteria, varicella zoster virus, herpes simplex virus, and mpox virus were all negative. A biopsy revealed suprabasilar acantholysis with follicular involvement in association with blister formation and inflammation. Direct immunofluorescence was positive for suprabasilar IgG and C3 deposition, consistent with pemphigus vulgaris (PV).

PV is an autoimmune bullous disease in which antibodies are directed against desmoglein 1 and 3 and less commonly, plakoglobin. There is likely a genetic predisposition. Medications that may induce pemphigus include penicillamine, nifedipine, or captopril.

Clinically, flaccid blistering lesions are present that may be cutaneous and/or mucosal. Bullae can progress to erosions and crusting, which then heal with pigment alteration but not scarring. The most commonly affected sites are the mouth, intertriginous areas, face, and neck. Mucosal lesions may involve the lips, esophagus, conjunctiva, and genitals.

BiluMartin_Donna_FLORIDA_web.jpg
%3Cp%3EDr.%20Donna%20Bilu%20Martin%3C%2Fp%3E


Biopsy for histology and direct immunofluorescence is important in distinguishing between PV and other blistering disorders. Up to 75% of patients with active disease also have a positive indirect immunofluorescence with circulating IgG.

Treatment is generally immunosuppressive. Systemic therapy usually begins with prednisone and then is transitioned to a steroid-sparing agent such as mycophenolate mofetil. Other steroid-sparing agents include azathioprine, methotrexate, cyclophosphamide, and intravenous immunoglobulin. Secondary infections are possible and should be treated. Topical therapies aimed at reducing pain, especially in mucosal lesions, can be beneficial.

This case and the photos are from Dr. Bilu Martin.

Dr. Bilu Martin is a board-certified dermatologist in private practice at Premier Dermatology, MD, in Aventura, Fla. More diagnostic cases are available at mdedge.com/dermatology. To submit a case for possible publication, send an email to dermnews@mdedge.com.

Publications
Topics
Sections

Cultures for bacteria, varicella zoster virus, herpes simplex virus, and mpox virus were all negative. A biopsy revealed suprabasilar acantholysis with follicular involvement in association with blister formation and inflammation. Direct immunofluorescence was positive for suprabasilar IgG and C3 deposition, consistent with pemphigus vulgaris (PV).

PV is an autoimmune bullous disease in which antibodies are directed against desmoglein 1 and 3 and less commonly, plakoglobin. There is likely a genetic predisposition. Medications that may induce pemphigus include penicillamine, nifedipine, or captopril.

Clinically, flaccid blistering lesions are present that may be cutaneous and/or mucosal. Bullae can progress to erosions and crusting, which then heal with pigment alteration but not scarring. The most commonly affected sites are the mouth, intertriginous areas, face, and neck. Mucosal lesions may involve the lips, esophagus, conjunctiva, and genitals.

BiluMartin_Donna_FLORIDA_web.jpg
%3Cp%3EDr.%20Donna%20Bilu%20Martin%3C%2Fp%3E


Biopsy for histology and direct immunofluorescence is important in distinguishing between PV and other blistering disorders. Up to 75% of patients with active disease also have a positive indirect immunofluorescence with circulating IgG.

Treatment is generally immunosuppressive. Systemic therapy usually begins with prednisone and then is transitioned to a steroid-sparing agent such as mycophenolate mofetil. Other steroid-sparing agents include azathioprine, methotrexate, cyclophosphamide, and intravenous immunoglobulin. Secondary infections are possible and should be treated. Topical therapies aimed at reducing pain, especially in mucosal lesions, can be beneficial.

This case and the photos are from Dr. Bilu Martin.

Dr. Bilu Martin is a board-certified dermatologist in private practice at Premier Dermatology, MD, in Aventura, Fla. More diagnostic cases are available at mdedge.com/dermatology. To submit a case for possible publication, send an email to dermnews@mdedge.com.

Cultures for bacteria, varicella zoster virus, herpes simplex virus, and mpox virus were all negative. A biopsy revealed suprabasilar acantholysis with follicular involvement in association with blister formation and inflammation. Direct immunofluorescence was positive for suprabasilar IgG and C3 deposition, consistent with pemphigus vulgaris (PV).

PV is an autoimmune bullous disease in which antibodies are directed against desmoglein 1 and 3 and less commonly, plakoglobin. There is likely a genetic predisposition. Medications that may induce pemphigus include penicillamine, nifedipine, or captopril.

Clinically, flaccid blistering lesions are present that may be cutaneous and/or mucosal. Bullae can progress to erosions and crusting, which then heal with pigment alteration but not scarring. The most commonly affected sites are the mouth, intertriginous areas, face, and neck. Mucosal lesions may involve the lips, esophagus, conjunctiva, and genitals.

BiluMartin_Donna_FLORIDA_web.jpg
%3Cp%3EDr.%20Donna%20Bilu%20Martin%3C%2Fp%3E


Biopsy for histology and direct immunofluorescence is important in distinguishing between PV and other blistering disorders. Up to 75% of patients with active disease also have a positive indirect immunofluorescence with circulating IgG.

Treatment is generally immunosuppressive. Systemic therapy usually begins with prednisone and then is transitioned to a steroid-sparing agent such as mycophenolate mofetil. Other steroid-sparing agents include azathioprine, methotrexate, cyclophosphamide, and intravenous immunoglobulin. Secondary infections are possible and should be treated. Topical therapies aimed at reducing pain, especially in mucosal lesions, can be beneficial.

This case and the photos are from Dr. Bilu Martin.

Dr. Bilu Martin is a board-certified dermatologist in private practice at Premier Dermatology, MD, in Aventura, Fla. More diagnostic cases are available at mdedge.com/dermatology. To submit a case for possible publication, send an email to dermnews@mdedge.com.

Publications
Publications
Topics
Article Type
Sections
Teambase XML
<?xml version="1.0" encoding="UTF-8"?>
<!--$RCSfile: InCopy_agile.xsl,v $ $Revision: 1.35 $-->
<!--$RCSfile: drupal.xsl,v $ $Revision: 1.7 $-->
<root generator="drupal.xsl" gversion="1.7"> <header> <fileName>164172</fileName> <TBEID>0C04B032.SIG</TBEID> <TBUniqueIdentifier>MD_0C04B032</TBUniqueIdentifier> <newsOrJournal>News</newsOrJournal> <publisherName>Frontline Medical Communications</publisherName> <storyname>August Make the Dx</storyname> <articleType>353</articleType> <TBLocation>QC Done-All Pubs</TBLocation> <QCDate>20230714T151520</QCDate> <firstPublished>20230807T120358</firstPublished> <LastPublished>20230807T120358</LastPublished> <pubStatus qcode="stat:"/> <embargoDate/> <killDate/> <CMSDate>20230807T120358</CMSDate> <articleSource/> <facebookInfo/> <meetingNumber/> <byline>Donna Bilu Martin</byline> <bylineText>DONNA BILU MARTIN, MD</bylineText> <bylineFull>DONNA BILU MARTIN, MD</bylineFull> <bylineTitleText/> <USOrGlobal/> <wireDocType/> <newsDocType>Column</newsDocType> <journalDocType/> <linkLabel/> <pageRange/> <citation/> <quizID/> <indexIssueDate/> <itemClass qcode="ninat:text"/> <provider qcode="provider:imng"> <name>IMNG Medical Media</name> <rightsInfo> <copyrightHolder> <name>Frontline Medical News</name> </copyrightHolder> <copyrightNotice>Copyright (c) 2015 Frontline Medical News, a Frontline Medical Communications Inc. company. All rights reserved. This material may not be published, broadcast, copied, or otherwise reproduced or distributed without the prior written permission of Frontline Medical Communications Inc.</copyrightNotice> </rightsInfo> </provider> <abstract/> <metaDescription>PV is an autoimmune bullous disease in which antibodies are directed against desmoglein 1 and 3 and less commonly, plakoglobin</metaDescription> <articlePDF/> <teaserImage>296451</teaserImage> <teaser>Biopsy for histology and direct immunofluorescence is important in distinguishing between PV and other blistering disorders. </teaser> <title>Pemphigus vulgaris</title> <deck/> <disclaimer/> <AuthorList/> <articleURL/> <doi/> <pubMedID/> <publishXMLStatus/> <publishXMLVersion>1</publishXMLVersion> <useEISSN>0</useEISSN> <urgency/> <pubPubdateYear/> <pubPubdateMonth/> <pubPubdateDay/> <pubVolume/> <pubNumber/> <wireChannels/> <primaryCMSID/> <CMSIDs/> <keywords/> <seeAlsos/> <publications_g> <publicationData> <publicationCode>skin</publicationCode> <pubIssueName/> <pubArticleType/> <pubTopics/> <pubCategories/> <pubSections/> </publicationData> <publicationData> <publicationCode>fp</publicationCode> <pubIssueName/> <pubArticleType/> <pubTopics/> <pubCategories/> <pubSections/> </publicationData> <publicationData> <publicationCode>im</publicationCode> <pubIssueName/> <pubArticleType/> <pubTopics/> <pubCategories/> <pubSections/> </publicationData> </publications_g> <publications> <term canonical="true">13</term> <term>15</term> <term>21</term> </publications> <sections> <term>52</term> <term canonical="true">87</term> </sections> <topics> <term>39212</term> <term canonical="true">29134</term> <term>203</term> </topics> <links> <link> <itemClass qcode="ninat:picture"/> <altRep contenttype="image/jpeg">images/24011fee.jpg</altRep> <description role="drol:caption"/> <description role="drol:credit"/> </link> </links> </header> <itemSet> <newsItem> <itemMeta> <itemRole>Main</itemRole> <itemClass>text</itemClass> <title>Pemphigus vulgaris</title> <deck/> </itemMeta> <itemContent> <p>Cultures for bacteria, varicella zoster virus, herpes simplex virus, and mpox virus were all negative. A biopsy revealed suprabasilar acantholysis with follicular involvement in association with blister formation and inflammation. Direct immunofluorescence was positive for suprabasilar IgG and C<sub>3 </sub>deposition, consistent with pemphigus vulgaris (PV).</p> <p><span class="tag metaDescription">PV is an autoimmune bullous disease in which antibodies are directed against desmoglein 1 and 3 and less commonly, plakoglobin</span>. There is likely a genetic predisposition. Medications that may induce pemphigus include <span class="Hyperlink"><a href="https://reference.medscape.com/drug/cuprimine-depen-penicillamine-343728">penicillamine</a></span>, <span class="Hyperlink"><a href="https://reference.medscape.com/drug/procardia-xl-nifedipine-342378">nifedipine</a></span>, or <span class="Hyperlink"><a href="https://reference.medscape.com/drug/capoten-captoril-captopril-342315">captopril</a></span>. <br/><br/>[[{"fid":"296451","view_mode":"medstat_image_flush_right","fields":{"format":"medstat_image_flush_right","field_file_image_alt_text[und][0][value]":"","field_file_image_credit[und][0][value]":"","field_file_image_caption[und][0][value]":""},"type":"media","attributes":{"class":"media-element file-medstat_image_flush_right"}}]]Clinically, flaccid blistering lesions are present that may be cutaneous and/or mucosal. Bullae can progress to erosions and crusting, which then heal with pigment alteration but not scarring. The most commonly affected sites are the mouth, intertriginous areas, face, and neck. Mucosal lesions may involve the lips, esophagus, conjunctiva, and genitals. <br/><br/>Biopsy for histology and direct immunofluorescence is important in distinguishing between PV and other blistering disorders. Up to 75% of patients with active disease also have a positive indirect immunofluorescence with circulating IgG. <br/><br/>Treatment is generally immunosuppressive. Systemic therapy usually begins with prednisone and then is transitioned to a steroid-sparing agent such as <span class="Hyperlink"><a href="https://reference.medscape.com/drug/cellcept-myfortic-mycophenolate-343209">mycophenolate mofetil</a></span>. Other steroid-sparing agents include azathioprine, methotrexate, cyclophosphamide, and intravenous immunoglobulin. Secondary infections are possible and should be treated. Topical therapies aimed at reducing pain, especially in mucosal lesions, can be beneficial. <br/><br/>This case and the photos are from Dr. Bilu Martin.<span class="end"/></p> <p> <em>Dr. Bilu Martin is a board-certified dermatologist in private practice at Premier Dermatology, MD, in Aventura, Fla. More diagnostic cases are available at <span class="Hyperlink"><a href="http://mdedge.com/dermatology">mdedge.com/dermatology</a></span>. To submit a case for possible publication, send an email to <span class="Hyperlink"><a href="mailto:dermnews%40mdedge.com?subject=">dermnews@mdedge.com</a></span>.</em> </p> </itemContent> </newsItem> <newsItem> <itemMeta> <itemRole>teaser</itemRole> <itemClass>text</itemClass> <title/> <deck/> </itemMeta> <itemContent> </itemContent> </newsItem> </itemSet></root>
Questionnaire Body

164172PV1LFT_web.jpg
A 45-year-old White woman with no significant medical history presented with a 1-month history of lesions on the nose and right cheek. She did an online teleheath visit with primary care and was prescribed acyclovir with no improvement. She subsequently took cefadroxil with no improvement. On physical examination, erythematous papules, vesicles, and erosions with an annular crusted border were present on the nose and cheeks. The patient denied any oral, mucosal, or genital lesions. The patient had no systemic symptoms.

Disallow All Ads
Content Gating
No Gating (article Unlocked/Free)
Alternative CME
Disqus Comments
Default
Use ProPublica
Hide sidebar & use full width
render the right sidebar.
Conference Recap Checkbox
Not Conference Recap
Clinical Edge
Display the Slideshow in this Article
Medscape Article
Display survey writer
Reuters content
Disable Inline Native ads
WebMD Article

A healthy 36-year-old female presented with 4 days of itchy lesions on the right upper extremity

Article Type
Changed
Fri, 05/12/2023 - 16:01

In this patient, bacterial and viral cultures were taken and varicella zoster virus (VZV) was isolated. Additionally, Orthopox DNA by PCR and Monkeypox (mpox) virus DNA by PCR were detected. Herpes simplex virus and bacterial viral cultures were negative. Valacyclovir was started at the time of presentation and the patient’s lesions resolved without sequelae.

Mpox is a zoonotic double-stranded DNA virus that is part of the Orthopoxvirus family, including the West African and Central African variants. This disease presents similarly to smallpox, so most mpox research was conducted around the time smallpox was eradicated. It was not until 1970, when the disease was isolated from a patient with suspected smallpox in the Democratic Republic of the Congo (DRC), that human mpox was considered a distinct disease. An epidemic outbreak in the United States occurred in 2003 related to infected prairie dogs, and travel-related outbreaks have been more recently reported up until May 2022, in which mpox was reported in nonendemic areas including North America, Europe, and Australia. Most cases in this outbreak occurred in men who have sex with men (MSM), but this is not always the case, and mpox is not necessarily considered a sexually transmitted infection. Mpox presents similarly to smallpox and VZV, so using laboratory tests is important in diagnosing and tracking this disease.

Although it is not easily transmitted, the disease can spread through bodily secretions both directly and indirectly. Mpox typically begins with a prodrome that includes fever, headache, myalgia, and fatigue. This is followed by lymphadenopathy that precedes and coincides with rash development. The lymph nodes are firm, tender, may be painful, and are a defining factor in presentation that differs from smallpox and varicella. The rash typically starts on the face, then presents on the body in a centrifugal distribution. However, cases related to sexual transmission present with anogenital lesions. The lesions are characterized by a progression from maculopapular to vesiculopustular, and can vary widely in quantity.

Notably, individuals are contagious from the onset of the prodrome until the lesions have scabbed over and fallen off. The eruptive nature of the later lesions poses a threat of secondary infection, and is often accompanied by a second febrile period that signifies deterioration of the patient’s condition. Other signs of secondary infection are variable and include pulmonary symptoms, vomiting, diarrhea, ocular infections, and in rare cases, encephalitis. These sequelae are more common in unvaccinated and immunocompromised individuals. Long-term complications of mpox include pitted scarring from cutaneous lesions with children being more susceptible to severe disease. The mortality rate for the disease is very low. (As of May 10, 2023, there have been 30,395 mpox cases reported in the United States, and 42 deaths, according to the Centers for Disease Control and Prevention.)

There are a variety of diagnostic tests that can aid in mpox identification, but they are most strongly supported when combined with clinical and epidemiological data. The best, least invasive method includes collection of lesion exudate or crust on a swab, and viral DNA is best preserved by keeping the specimen in a cool, dry, and dark environment. PCR is considered the standard, and electron microscopy and immunohistochemistry are valid tests, but all modalities require sophisticated technicians with the proper laboratory equipment. This is limiting because many cases present in underserved areas that lack the facilities for proper, real-time analysis. Antigen and antibody-based tests can be used, but cross-reactivity of other orthopoxviridae limits confirmation of mpox infection. Vaccination status, history and location must be considered.

Vaccination is the chief form of prevention for mpox, although it is not considered entirely protective. Smallpox vaccination provides protection, but widespread administration of the vaccine is no longer practiced, and an estimated 70% of the global population is no longer vaccinated. Vaccination is recommended for anyone at risk of exposure, but as this is a live, attenuated vaccine, the immune status of the patient is important to keep in mind. Tecovirimat and other antiviral medications including cidofovir and brincidofovir may be considered in severe cases.

This case is unique as our patient, who had no known risk factors for mpox, presented with mpox and VZV, simultaneously. Although clinical presentation and epidemiological patterns between these diseases differ, there have been a limited number of cases of coinfection reported in the literature, mainly in the DRC where mpox is endemic. Diagnosis must be made by separate laboratory tests and there are differences in presentation between independent and coinfection for these viruses. Notably, patients with mpox/VZV coinfection may be less likely to present with lesions on the face, thorax, arms, palms, and soles than those with only mpox but experience a higher lesion burden than those afflicted by only VZV. Coinfection may be related to reactivation of dormant VZV, or increased susceptibility to secondary infection when infected with one virus.

This case and photo were submitted by Lucas Shapiro, BS, of the Dr. Kiran C. Patel College of Osteopathic Medicine at Nova Southeastern University, Fort Lauderdale, Fla., and Donna Bilu Martin, MD.

Dr. Bilu Martin is a board-certified dermatologist in private practice at Premier Dermatology, MD, in Aventura, Fla. More diagnostic cases are available at mdedge.com/dermatology. To submit a case for possible publication, send an email to dermnews@mdedge.com.

References

1. Macneil A et al. Clin Infect Dis. 2009 Jan 1;48(1):e6-8.

2. Di Gennaro F et al. Microorganisms. 2022 Aug 12;10(8):1633.

3. Hughes CM et al. Am J Trop Med Hyg. 2020 Dec 7;104(2):604-11.

Publications
Topics
Sections

In this patient, bacterial and viral cultures were taken and varicella zoster virus (VZV) was isolated. Additionally, Orthopox DNA by PCR and Monkeypox (mpox) virus DNA by PCR were detected. Herpes simplex virus and bacterial viral cultures were negative. Valacyclovir was started at the time of presentation and the patient’s lesions resolved without sequelae.

Mpox is a zoonotic double-stranded DNA virus that is part of the Orthopoxvirus family, including the West African and Central African variants. This disease presents similarly to smallpox, so most mpox research was conducted around the time smallpox was eradicated. It was not until 1970, when the disease was isolated from a patient with suspected smallpox in the Democratic Republic of the Congo (DRC), that human mpox was considered a distinct disease. An epidemic outbreak in the United States occurred in 2003 related to infected prairie dogs, and travel-related outbreaks have been more recently reported up until May 2022, in which mpox was reported in nonendemic areas including North America, Europe, and Australia. Most cases in this outbreak occurred in men who have sex with men (MSM), but this is not always the case, and mpox is not necessarily considered a sexually transmitted infection. Mpox presents similarly to smallpox and VZV, so using laboratory tests is important in diagnosing and tracking this disease.

Although it is not easily transmitted, the disease can spread through bodily secretions both directly and indirectly. Mpox typically begins with a prodrome that includes fever, headache, myalgia, and fatigue. This is followed by lymphadenopathy that precedes and coincides with rash development. The lymph nodes are firm, tender, may be painful, and are a defining factor in presentation that differs from smallpox and varicella. The rash typically starts on the face, then presents on the body in a centrifugal distribution. However, cases related to sexual transmission present with anogenital lesions. The lesions are characterized by a progression from maculopapular to vesiculopustular, and can vary widely in quantity.

Notably, individuals are contagious from the onset of the prodrome until the lesions have scabbed over and fallen off. The eruptive nature of the later lesions poses a threat of secondary infection, and is often accompanied by a second febrile period that signifies deterioration of the patient’s condition. Other signs of secondary infection are variable and include pulmonary symptoms, vomiting, diarrhea, ocular infections, and in rare cases, encephalitis. These sequelae are more common in unvaccinated and immunocompromised individuals. Long-term complications of mpox include pitted scarring from cutaneous lesions with children being more susceptible to severe disease. The mortality rate for the disease is very low. (As of May 10, 2023, there have been 30,395 mpox cases reported in the United States, and 42 deaths, according to the Centers for Disease Control and Prevention.)

There are a variety of diagnostic tests that can aid in mpox identification, but they are most strongly supported when combined with clinical and epidemiological data. The best, least invasive method includes collection of lesion exudate or crust on a swab, and viral DNA is best preserved by keeping the specimen in a cool, dry, and dark environment. PCR is considered the standard, and electron microscopy and immunohistochemistry are valid tests, but all modalities require sophisticated technicians with the proper laboratory equipment. This is limiting because many cases present in underserved areas that lack the facilities for proper, real-time analysis. Antigen and antibody-based tests can be used, but cross-reactivity of other orthopoxviridae limits confirmation of mpox infection. Vaccination status, history and location must be considered.

Vaccination is the chief form of prevention for mpox, although it is not considered entirely protective. Smallpox vaccination provides protection, but widespread administration of the vaccine is no longer practiced, and an estimated 70% of the global population is no longer vaccinated. Vaccination is recommended for anyone at risk of exposure, but as this is a live, attenuated vaccine, the immune status of the patient is important to keep in mind. Tecovirimat and other antiviral medications including cidofovir and brincidofovir may be considered in severe cases.

This case is unique as our patient, who had no known risk factors for mpox, presented with mpox and VZV, simultaneously. Although clinical presentation and epidemiological patterns between these diseases differ, there have been a limited number of cases of coinfection reported in the literature, mainly in the DRC where mpox is endemic. Diagnosis must be made by separate laboratory tests and there are differences in presentation between independent and coinfection for these viruses. Notably, patients with mpox/VZV coinfection may be less likely to present with lesions on the face, thorax, arms, palms, and soles than those with only mpox but experience a higher lesion burden than those afflicted by only VZV. Coinfection may be related to reactivation of dormant VZV, or increased susceptibility to secondary infection when infected with one virus.

This case and photo were submitted by Lucas Shapiro, BS, of the Dr. Kiran C. Patel College of Osteopathic Medicine at Nova Southeastern University, Fort Lauderdale, Fla., and Donna Bilu Martin, MD.

Dr. Bilu Martin is a board-certified dermatologist in private practice at Premier Dermatology, MD, in Aventura, Fla. More diagnostic cases are available at mdedge.com/dermatology. To submit a case for possible publication, send an email to dermnews@mdedge.com.

References

1. Macneil A et al. Clin Infect Dis. 2009 Jan 1;48(1):e6-8.

2. Di Gennaro F et al. Microorganisms. 2022 Aug 12;10(8):1633.

3. Hughes CM et al. Am J Trop Med Hyg. 2020 Dec 7;104(2):604-11.

In this patient, bacterial and viral cultures were taken and varicella zoster virus (VZV) was isolated. Additionally, Orthopox DNA by PCR and Monkeypox (mpox) virus DNA by PCR were detected. Herpes simplex virus and bacterial viral cultures were negative. Valacyclovir was started at the time of presentation and the patient’s lesions resolved without sequelae.

Mpox is a zoonotic double-stranded DNA virus that is part of the Orthopoxvirus family, including the West African and Central African variants. This disease presents similarly to smallpox, so most mpox research was conducted around the time smallpox was eradicated. It was not until 1970, when the disease was isolated from a patient with suspected smallpox in the Democratic Republic of the Congo (DRC), that human mpox was considered a distinct disease. An epidemic outbreak in the United States occurred in 2003 related to infected prairie dogs, and travel-related outbreaks have been more recently reported up until May 2022, in which mpox was reported in nonendemic areas including North America, Europe, and Australia. Most cases in this outbreak occurred in men who have sex with men (MSM), but this is not always the case, and mpox is not necessarily considered a sexually transmitted infection. Mpox presents similarly to smallpox and VZV, so using laboratory tests is important in diagnosing and tracking this disease.

Although it is not easily transmitted, the disease can spread through bodily secretions both directly and indirectly. Mpox typically begins with a prodrome that includes fever, headache, myalgia, and fatigue. This is followed by lymphadenopathy that precedes and coincides with rash development. The lymph nodes are firm, tender, may be painful, and are a defining factor in presentation that differs from smallpox and varicella. The rash typically starts on the face, then presents on the body in a centrifugal distribution. However, cases related to sexual transmission present with anogenital lesions. The lesions are characterized by a progression from maculopapular to vesiculopustular, and can vary widely in quantity.

Notably, individuals are contagious from the onset of the prodrome until the lesions have scabbed over and fallen off. The eruptive nature of the later lesions poses a threat of secondary infection, and is often accompanied by a second febrile period that signifies deterioration of the patient’s condition. Other signs of secondary infection are variable and include pulmonary symptoms, vomiting, diarrhea, ocular infections, and in rare cases, encephalitis. These sequelae are more common in unvaccinated and immunocompromised individuals. Long-term complications of mpox include pitted scarring from cutaneous lesions with children being more susceptible to severe disease. The mortality rate for the disease is very low. (As of May 10, 2023, there have been 30,395 mpox cases reported in the United States, and 42 deaths, according to the Centers for Disease Control and Prevention.)

There are a variety of diagnostic tests that can aid in mpox identification, but they are most strongly supported when combined with clinical and epidemiological data. The best, least invasive method includes collection of lesion exudate or crust on a swab, and viral DNA is best preserved by keeping the specimen in a cool, dry, and dark environment. PCR is considered the standard, and electron microscopy and immunohistochemistry are valid tests, but all modalities require sophisticated technicians with the proper laboratory equipment. This is limiting because many cases present in underserved areas that lack the facilities for proper, real-time analysis. Antigen and antibody-based tests can be used, but cross-reactivity of other orthopoxviridae limits confirmation of mpox infection. Vaccination status, history and location must be considered.

Vaccination is the chief form of prevention for mpox, although it is not considered entirely protective. Smallpox vaccination provides protection, but widespread administration of the vaccine is no longer practiced, and an estimated 70% of the global population is no longer vaccinated. Vaccination is recommended for anyone at risk of exposure, but as this is a live, attenuated vaccine, the immune status of the patient is important to keep in mind. Tecovirimat and other antiviral medications including cidofovir and brincidofovir may be considered in severe cases.

This case is unique as our patient, who had no known risk factors for mpox, presented with mpox and VZV, simultaneously. Although clinical presentation and epidemiological patterns between these diseases differ, there have been a limited number of cases of coinfection reported in the literature, mainly in the DRC where mpox is endemic. Diagnosis must be made by separate laboratory tests and there are differences in presentation between independent and coinfection for these viruses. Notably, patients with mpox/VZV coinfection may be less likely to present with lesions on the face, thorax, arms, palms, and soles than those with only mpox but experience a higher lesion burden than those afflicted by only VZV. Coinfection may be related to reactivation of dormant VZV, or increased susceptibility to secondary infection when infected with one virus.

This case and photo were submitted by Lucas Shapiro, BS, of the Dr. Kiran C. Patel College of Osteopathic Medicine at Nova Southeastern University, Fort Lauderdale, Fla., and Donna Bilu Martin, MD.

Dr. Bilu Martin is a board-certified dermatologist in private practice at Premier Dermatology, MD, in Aventura, Fla. More diagnostic cases are available at mdedge.com/dermatology. To submit a case for possible publication, send an email to dermnews@mdedge.com.

References

1. Macneil A et al. Clin Infect Dis. 2009 Jan 1;48(1):e6-8.

2. Di Gennaro F et al. Microorganisms. 2022 Aug 12;10(8):1633.

3. Hughes CM et al. Am J Trop Med Hyg. 2020 Dec 7;104(2):604-11.

Publications
Publications
Topics
Article Type
Sections
Teambase XML
<?xml version="1.0" encoding="UTF-8"?>
<!--$RCSfile: InCopy_agile.xsl,v $ $Revision: 1.35 $-->
<!--$RCSfile: drupal.xsl,v $ $Revision: 1.7 $-->
<root generator="drupal.xsl" gversion="1.7"> <header> <fileName>163381</fileName> <TBEID>0C04A00B.SIG</TBEID> <TBUniqueIdentifier>MD_0C04A00B</TBUniqueIdentifier> <newsOrJournal>News</newsOrJournal> <publisherName>Frontline Medical Communications</publisherName> <storyname>June Make the Diagnosis</storyname> <articleType>353</articleType> <TBLocation>QC Done-All Pubs</TBLocation> <QCDate>20230512T153420</QCDate> <firstPublished>20230512T155506</firstPublished> <LastPublished>20230512T155506</LastPublished> <pubStatus qcode="stat:"/> <embargoDate/> <killDate/> <CMSDate>20230512T155506</CMSDate> <articleSource/> <facebookInfo/> <meetingNumber/> <byline>Bilu Martin and Shapiro</byline> <bylineText>LUCAS SHAPIRO, BS, AND DONNA BILU MARTIN, MD</bylineText> <bylineFull>LUCAS SHAPIRO, BS, AND DONNA BILU MARTIN, MD</bylineFull> <bylineTitleText/> <USOrGlobal/> <wireDocType/> <newsDocType>Column</newsDocType> <journalDocType/> <linkLabel/> <pageRange/> <citation/> <quizID/> <indexIssueDate/> <itemClass qcode="ninat:text"/> <provider qcode="provider:imng"> <name>IMNG Medical Media</name> <rightsInfo> <copyrightHolder> <name>Frontline Medical News</name> </copyrightHolder> <copyrightNotice>Copyright (c) 2015 Frontline Medical News, a Frontline Medical Communications Inc. company. All rights reserved. This material may not be published, broadcast, copied, or otherwise reproduced or distributed without the prior written permission of Frontline Medical Communications Inc.</copyrightNotice> </rightsInfo> </provider> <abstract/> <metaDescription>In this patient, bacterial and viral cultures were taken and varicella zoster virus (VZV) was isolated.</metaDescription> <articlePDF/> <teaserImage/> <title>Varicella zoster and monkeypox (mpox) virus</title> <deck/> <disclaimer/> <AuthorList/> <articleURL/> <doi/> <pubMedID/> <publishXMLStatus/> <publishXMLVersion>1</publishXMLVersion> <useEISSN>0</useEISSN> <urgency/> <pubPubdateYear/> <pubPubdateMonth/> <pubPubdateDay/> <pubVolume/> <pubNumber/> <wireChannels/> <primaryCMSID/> <CMSIDs/> <keywords/> <seeAlsos/> <publications_g> <publicationData> <publicationCode>skin</publicationCode> <pubIssueName/> <pubArticleType/> <pubTopics/> <pubCategories/> <pubSections/> </publicationData> <publicationData> <publicationCode>im</publicationCode> <pubIssueName/> <pubArticleType/> <pubTopics/> <pubCategories/> <pubSections/> </publicationData> <publicationData> <publicationCode>fp</publicationCode> <pubIssueName/> <pubArticleType/> <pubTopics/> <pubCategories/> <pubSections/> </publicationData> </publications_g> <publications> <term canonical="true">13</term> <term>21</term> <term>15</term> </publications> <sections> <term>52</term> <term canonical="true">87</term> </sections> <topics> <term>39212</term> <term canonical="true">234</term> <term>203</term> </topics> <links/> </header> <itemSet> <newsItem> <itemMeta> <itemRole>Main</itemRole> <itemClass>text</itemClass> <title>Varicella zoster and monkeypox (mpox) virus</title> <deck/> </itemMeta> <itemContent> <p><span class="tag metaDescription">In this patient, bacterial and viral cultures were taken and varicella zoster virus (VZV) was isolated.</span> Additionally, Orthopox DNA by PCR and Monkeypox (mpox) virus DNA by PCR were detected. Herpes simplex virus and bacterial viral cultures were negative. Valacyclovir was started at the time of presentation and the patient’s lesions resolved without sequelae.</p> <p>Mpox is a zoonotic double-stranded DNA virus that is part of the Orthopoxvirus family, including the West African and Central African variants. This disease presents similarly to smallpox, so most mpox research was conducted around the time smallpox was eradicated. It was not until 1970, when the disease was isolated from a patient with suspected smallpox in the Democratic Republic of the Congo (DRC), that human mpox was considered a distinct disease. An epidemic outbreak in the United States occurred in 2003 related to infected prairie dogs, and travel-related outbreaks have been more recently reported up until May 2022, in which mpox was reported in nonendemic areas including North America, Europe, and Australia. Most cases in this outbreak occurred in men who have sex with men (MSM), but this is not always the case, and mpox is not necessarily considered a sexually transmitted infection. Mpox presents similarly to smallpox and VZV, so using laboratory tests is important in diagnosing and tracking this disease. <br/><br/>Although it is not easily transmitted, the disease can spread through bodily secretions both directly and indirectly. Mpox typically begins with a prodrome that includes fever, headache, myalgia, and fatigue. This is followed by lymphadenopathy that precedes and coincides with rash development. The lymph nodes are firm, tender, may be painful, and are a defining factor in presentation that differs from smallpox and varicella. The rash typically starts on the face, then presents on the body in a centrifugal distribution. However, cases related to sexual transmission present with anogenital lesions. The lesions are characterized by a progression from maculopapular to vesiculopustular, and can vary widely in quantity. <br/><br/>Notably, individuals are contagious from the onset of the prodrome until the lesions have scabbed over and fallen off. The eruptive nature of the later lesions poses a threat of secondary infection, and is often accompanied by a second febrile period that signifies deterioration of the patient’s condition. Other signs of secondary infection are variable and include pulmonary symptoms, vomiting, diarrhea, ocular infections, and in rare cases, encephalitis. These sequelae are more common in unvaccinated and immunocompromised individuals. Long-term complications of mpox include pitted scarring from cutaneous lesions with children being more susceptible to severe disease. The mortality rate for the disease is very low. (As of May 10, 2023, there have been 30,395 mpox cases reported in the United States, and 42 deaths, <span class="Hyperlink"><a href="https://www.cdc.gov/poxvirus/mpox/response/2022/us-map.html">according to the Centers for Disease Control and Prevention</a></span>.)<br/><br/>There are a variety of diagnostic tests that can aid in mpox identification, but they are most strongly supported when combined with clinical and epidemiological data. The best, least invasive method includes collection of lesion exudate or crust on a swab, and viral DNA is best preserved by keeping the specimen in a cool, dry, and dark environment. PCR is considered the standard, and electron microscopy and immunohistochemistry are valid tests, but all modalities require sophisticated technicians with the proper laboratory equipment. This is limiting because many cases present in underserved areas that lack the facilities for proper, real-time analysis. Antigen and antibody-based tests can be used, but cross-reactivity of other orthopoxviridae limits confirmation of mpox infection. Vaccination status, history and location must be considered.<br/><br/>Vaccination is the chief form of prevention for mpox, although it is not considered entirely protective. Smallpox vaccination provides protection, but widespread administration of the vaccine is no longer practiced, and an estimated 70% of the global population is no longer vaccinated. Vaccination is recommended for anyone at risk of exposure, but as this is a live, attenuated vaccine, the immune status of the patient is important to keep in mind. Tecovirimat and other antiviral medications including cidofovir and brincidofovir may be considered in severe cases. <br/><br/>This case is unique as our patient, who had no known risk factors for mpox, presented with mpox and VZV, simultaneously. Although clinical presentation and epidemiological patterns between these diseases differ, there have been a limited number of cases of coinfection reported in the literature, mainly in the DRC where mpox is endemic. Diagnosis must be made by separate laboratory tests and there are differences in presentation between independent and coinfection for these viruses. Notably, patients with mpox/VZV coinfection may be less likely to present with lesions on the face, thorax, arms, palms, and soles than those with only mpox but experience a higher lesion burden than those afflicted by only VZV. Coinfection may be related to reactivation of dormant VZV, or increased susceptibility to secondary infection when infected with one virus. <br/><br/>This case and photo were submitted by Lucas Shapiro, BS, of the Dr. Kiran C. Patel College of Osteopathic Medicine at Nova Southeastern University, Fort Lauderdale, Fla., and Donna Bilu Martin, MD.<span class="end"/></p> <p> <em>Dr. Bilu Martin is a board-certified dermatologist in private practice at Premier Dermatology, MD, in Aventura, Fla. More diagnostic cases are available at <span class="Hyperlink"><a href="http://mdedge.com/dermatology">mdedge.com/dermatology</a></span>. To submit a case for possible publication, send an email to <span class="Hyperlink"><a href="mailto:dermnews%40mdedge.com?subject=">dermnews@mdedge.com</a></span>.</em> </p> <h2>References</h2> <p>1. Macneil A et al. <span class="Hyperlink"><a href="Clin Infect Dhttps://www.ncbi.nlm.nih.gov/pmc/articles/PMC5895105/is. 2009 Jan 1;48(1):e6-8">Clin Infect Dis. 2009 Jan 1;48(1):e6-8</a></span>.<br/><br/>2. Di Gennaro F et al. <span class="Hyperlink"><a href="https://www.ncbi.nlm.nih.gov/pmc/articles/PMC9416167/">Microorganisms. 2022 Aug 12;10(8):1633</a></span>. <br/><br/>3. Hughes CM et al. <span class="Hyperlink"><a href="https://www.ncbi.nlm.nih.gov/pmc/articles/PMC7866336/">Am J Trop Med Hyg. 2020 Dec 7;104(2):604-11</a></span>.</p> </itemContent> </newsItem> <newsItem> <itemMeta> <itemRole>teaser</itemRole> <itemClass>text</itemClass> <title/> <deck/> </itemMeta> <itemContent> <p>Mpox presents similarly to smallpox and varicella zoster, so using laboratory tests is important in diagnosing and tracking this disease.</p> </itemContent> </newsItem> </itemSet></root>
Questionnaire Body

163381_photo_web.jpg
A healthy 36-year-old female presented with 4 days of itchy lesions on the right upper extremity. She stated that she thought she was "bitten by an insect." On physical examination, six grouped vesicles were present with mild surrounding erythema. She had no systemic symptoms. She did have mild lymphadenopathy.

Disallow All Ads
Content Gating
No Gating (article Unlocked/Free)
Alternative CME
Disqus Comments
Default
Use ProPublica
Hide sidebar & use full width
render the right sidebar.
Conference Recap Checkbox
Not Conference Recap
Clinical Edge
Display the Slideshow in this Article
Medscape Article
Display survey writer
Reuters content
Disable Inline Native ads
WebMD Article

White male presents with pruritic, scaly, erythematous patches on his feet and left hand

Article Type
Changed
Thu, 03/09/2023 - 15:16

 

Two feet–one hand syndrome

Two feet–one hand syndrome is a common term used to describe tinea manuum on one hand with bilateral tinea pedis. This condition, also known as ringworm, is a fungal infection caused by a dermatophyte, and presents as a superficial annular or circular rash with a raised, scaly border.

Symptoms include dryness and itchiness, and the lesions may appear red-pink on lighter skin and gray-brown on darker skin types. Although these infections can arise in a variety of combinations, two feet–one hand syndrome occurs in about 60% of cases. Trichophyton rubrum is the most common agent.

Diagnosis is made by patient history, dermoscopic visualization, and staining of skin scraping with KOH or fungal culture. Dermatophytes prefer moist, warm environments, so this disease is prevalent in tropical conditions and associated with moist public areas such as locker rooms and showers. As a result, tinea pedis is also nicknamed “athlete’s foot” for its common presentation in athletes. The fungus spreads easily through contact and can survive on infected surfaces, so patients often self-inoculate by touching/scratching the affected area then touching another body part. Cautions that should be taken to avoid transmission include not sharing personal care products, washing the area and keeping it dry, and avoiding close, humid environments.

162413_hand_web.jpg

The syndrome is highly associated with onychomycosis, which can be more difficult to treat and often requires oral antifungals. Tinea manuum is commonly misdiagnosed as hand dermatitis or eczema and treated with topical steroids, which will exacerbate or flare the tinea.

Two feet–one hand syndrome can typically be treated with over-the-counter topical antifungal medications such as miconazole or clotrimazole. Topical ketoconazole may be prescribed, and oral terbinafine or itraconazole are used in more severe cases when a larger body surface area is affected or in immunocompromised patients.

162413_palm_web.jpg

This case and photo were submitted by Lucas Shapiro, BS, Nova Southeastern University, Davie, Fla.; Kiran C. Patel, Tampa Bay Regional Campus; and Dr. Bilu Martin.
 

Dr. Bilu Martin is a board-certified dermatologist in private practice at Premier Dermatology, MD, in Aventura, Fla. More diagnostic cases are available at mdedge.com/dermatology. To submit a case for possible publication, send an email to dermnews@mdedge.com.

BiluMartin_Donna_FLORIDA_web.jpg
Dr. Donna Bilu Martin

References

Cleveland Clinic. Tinea manuum: Symptoms, causes & treatment. 2022. https://my.clevelandclinic.org/health/diseases/24063-tinea-manuum.

Ugalde-Trejo NX et al. Curr Fungal Infect Rep. 2022 Nov 17. doi: 10.1007/s12281-022-00447-9.

Mizumoto J. Cureus. 2021 Dec 27;13(12):e20758.

Publications
Topics
Sections

 

Two feet–one hand syndrome

Two feet–one hand syndrome is a common term used to describe tinea manuum on one hand with bilateral tinea pedis. This condition, also known as ringworm, is a fungal infection caused by a dermatophyte, and presents as a superficial annular or circular rash with a raised, scaly border.

Symptoms include dryness and itchiness, and the lesions may appear red-pink on lighter skin and gray-brown on darker skin types. Although these infections can arise in a variety of combinations, two feet–one hand syndrome occurs in about 60% of cases. Trichophyton rubrum is the most common agent.

Diagnosis is made by patient history, dermoscopic visualization, and staining of skin scraping with KOH or fungal culture. Dermatophytes prefer moist, warm environments, so this disease is prevalent in tropical conditions and associated with moist public areas such as locker rooms and showers. As a result, tinea pedis is also nicknamed “athlete’s foot” for its common presentation in athletes. The fungus spreads easily through contact and can survive on infected surfaces, so patients often self-inoculate by touching/scratching the affected area then touching another body part. Cautions that should be taken to avoid transmission include not sharing personal care products, washing the area and keeping it dry, and avoiding close, humid environments.

162413_hand_web.jpg

The syndrome is highly associated with onychomycosis, which can be more difficult to treat and often requires oral antifungals. Tinea manuum is commonly misdiagnosed as hand dermatitis or eczema and treated with topical steroids, which will exacerbate or flare the tinea.

Two feet–one hand syndrome can typically be treated with over-the-counter topical antifungal medications such as miconazole or clotrimazole. Topical ketoconazole may be prescribed, and oral terbinafine or itraconazole are used in more severe cases when a larger body surface area is affected or in immunocompromised patients.

162413_palm_web.jpg

This case and photo were submitted by Lucas Shapiro, BS, Nova Southeastern University, Davie, Fla.; Kiran C. Patel, Tampa Bay Regional Campus; and Dr. Bilu Martin.
 

Dr. Bilu Martin is a board-certified dermatologist in private practice at Premier Dermatology, MD, in Aventura, Fla. More diagnostic cases are available at mdedge.com/dermatology. To submit a case for possible publication, send an email to dermnews@mdedge.com.

BiluMartin_Donna_FLORIDA_web.jpg
Dr. Donna Bilu Martin

References

Cleveland Clinic. Tinea manuum: Symptoms, causes & treatment. 2022. https://my.clevelandclinic.org/health/diseases/24063-tinea-manuum.

Ugalde-Trejo NX et al. Curr Fungal Infect Rep. 2022 Nov 17. doi: 10.1007/s12281-022-00447-9.

Mizumoto J. Cureus. 2021 Dec 27;13(12):e20758.

 

Two feet–one hand syndrome

Two feet–one hand syndrome is a common term used to describe tinea manuum on one hand with bilateral tinea pedis. This condition, also known as ringworm, is a fungal infection caused by a dermatophyte, and presents as a superficial annular or circular rash with a raised, scaly border.

Symptoms include dryness and itchiness, and the lesions may appear red-pink on lighter skin and gray-brown on darker skin types. Although these infections can arise in a variety of combinations, two feet–one hand syndrome occurs in about 60% of cases. Trichophyton rubrum is the most common agent.

Diagnosis is made by patient history, dermoscopic visualization, and staining of skin scraping with KOH or fungal culture. Dermatophytes prefer moist, warm environments, so this disease is prevalent in tropical conditions and associated with moist public areas such as locker rooms and showers. As a result, tinea pedis is also nicknamed “athlete’s foot” for its common presentation in athletes. The fungus spreads easily through contact and can survive on infected surfaces, so patients often self-inoculate by touching/scratching the affected area then touching another body part. Cautions that should be taken to avoid transmission include not sharing personal care products, washing the area and keeping it dry, and avoiding close, humid environments.

162413_hand_web.jpg

The syndrome is highly associated with onychomycosis, which can be more difficult to treat and often requires oral antifungals. Tinea manuum is commonly misdiagnosed as hand dermatitis or eczema and treated with topical steroids, which will exacerbate or flare the tinea.

Two feet–one hand syndrome can typically be treated with over-the-counter topical antifungal medications such as miconazole or clotrimazole. Topical ketoconazole may be prescribed, and oral terbinafine or itraconazole are used in more severe cases when a larger body surface area is affected or in immunocompromised patients.

162413_palm_web.jpg

This case and photo were submitted by Lucas Shapiro, BS, Nova Southeastern University, Davie, Fla.; Kiran C. Patel, Tampa Bay Regional Campus; and Dr. Bilu Martin.
 

Dr. Bilu Martin is a board-certified dermatologist in private practice at Premier Dermatology, MD, in Aventura, Fla. More diagnostic cases are available at mdedge.com/dermatology. To submit a case for possible publication, send an email to dermnews@mdedge.com.

BiluMartin_Donna_FLORIDA_web.jpg
Dr. Donna Bilu Martin

References

Cleveland Clinic. Tinea manuum: Symptoms, causes & treatment. 2022. https://my.clevelandclinic.org/health/diseases/24063-tinea-manuum.

Ugalde-Trejo NX et al. Curr Fungal Infect Rep. 2022 Nov 17. doi: 10.1007/s12281-022-00447-9.

Mizumoto J. Cureus. 2021 Dec 27;13(12):e20758.

Publications
Publications
Topics
Article Type
Sections
Teambase XML
<?xml version="1.0" encoding="UTF-8"?>
<!--$RCSfile: InCopy_agile.xsl,v $ $Revision: 1.35 $-->
<!--$RCSfile: drupal.xsl,v $ $Revision: 1.7 $-->
<root generator="drupal.xsl" gversion="1.7"> <header> <fileName>162413</fileName> <TBEID>0C048AAE.SIG</TBEID> <TBUniqueIdentifier>MD_0C048AAE</TBUniqueIdentifier> <newsOrJournal>News</newsOrJournal> <publisherName>Frontline Medical Communications</publisherName> <storyname>April Make the Dx</storyname> <articleType>353</articleType> <TBLocation>QC Done-All Pubs</TBLocation> <QCDate>20230309T090350</QCDate> <firstPublished>20230309T150904</firstPublished> <LastPublished>20230309T150904</LastPublished> <pubStatus qcode="stat:"/> <embargoDate/> <killDate/> <CMSDate>20230309T150904</CMSDate> <articleSource/> <facebookInfo/> <meetingNumber/> <byline>Donna Bilu Martin</byline> <bylineText>DONNA BILU MARTIN, MD</bylineText> <bylineFull>DONNA BILU MARTIN, MD</bylineFull> <bylineTitleText/> <USOrGlobal/> <wireDocType/> <newsDocType>Column</newsDocType> <journalDocType/> <linkLabel/> <pageRange/> <citation/> <quizID/> <indexIssueDate/> <itemClass qcode="ninat:text"/> <provider qcode="provider:imng"> <name>IMNG Medical Media</name> <rightsInfo> <copyrightHolder> <name>Frontline Medical News</name> </copyrightHolder> <copyrightNotice>Copyright (c) 2015 Frontline Medical News, a Frontline Medical Communications Inc. company. All rights reserved. This material may not be published, broadcast, copied, or otherwise reproduced or distributed without the prior written permission of Frontline Medical Communications Inc.</copyrightNotice> </rightsInfo> </provider> <abstract/> <metaDescription>Two feet–one hand syndrome is a common term used to describe tinea manuum on one hand with bilateral tinea pedis.</metaDescription> <articlePDF/> <teaserImage>293565</teaserImage> <teaser>Tinea manuum is commonly misdiagnosed as hand dermatitis or eczema and treated with topical steroids, which will exacerbate or flare the tinea. </teaser> <title/> <deck/> <disclaimer/> <AuthorList/> <articleURL/> <doi/> <pubMedID/> <publishXMLStatus/> <publishXMLVersion>1</publishXMLVersion> <useEISSN>0</useEISSN> <urgency/> <pubPubdateYear/> <pubPubdateMonth/> <pubPubdateDay/> <pubVolume/> <pubNumber/> <wireChannels/> <primaryCMSID/> <CMSIDs/> <keywords/> <seeAlsos/> <publications_g> <publicationData> <publicationCode>skin</publicationCode> <pubIssueName/> <pubArticleType/> <pubTopics/> <pubCategories/> <pubSections/> </publicationData> <publicationData> <publicationCode>im</publicationCode> <pubIssueName/> <pubArticleType/> <pubTopics/> <pubCategories/> <pubSections/> </publicationData> <publicationData> <publicationCode>fp</publicationCode> <pubIssueName/> <pubArticleType/> <pubTopics/> <pubCategories/> <pubSections/> </publicationData> </publications_g> <publications> <term canonical="true">13</term> <term>21</term> <term>15</term> </publications> <sections> <term>52</term> <term canonical="true">87</term> </sections> <topics> <term canonical="true">234</term> <term>39212</term> <term>203</term> </topics> <links> <link> <itemClass qcode="ninat:picture"/> <altRep contenttype="image/jpeg">images/24011a76.jpg</altRep> <description role="drol:caption"/> <description role="drol:credit">Dr. Donna Bilu Martin</description> </link> <link> <itemClass qcode="ninat:picture"/> <altRep contenttype="image/jpeg">images/24011a77.jpg</altRep> <description role="drol:caption"/> <description role="drol:credit">Dr. Donna Bilu Martin</description> </link> <link> <itemClass qcode="ninat:picture"/> <altRep contenttype="image/jpeg">images/24011a78.jpg</altRep> <description role="drol:caption"/> <description role="drol:credit">Dr. Donna Bilu Martin</description> </link> <link> <itemClass qcode="ninat:picture"/> <altRep contenttype="image/jpeg">images/2400f1a6.jpg</altRep> <description role="drol:caption">Dr. Donna Bilu Martin</description> <description role="drol:credit"/> </link> </links> </header> <itemSet> <newsItem> <itemMeta> <itemRole>Main</itemRole> <itemClass>text</itemClass> <title/> <deck/> </itemMeta> <itemContent> <h2>Two feet–one hand syndrome</h2> <p><span class="tag metaDescription">Two feet–one hand syndrome is a common term used to describe tinea manuum on one hand with bilateral tinea pedis.</span> This condition, also known as ringworm, is a fungal infection caused by a dermatophyte, and presents as a superficial annular or circular rash with a raised, scaly border. </p> <p>Symptoms include dryness and itchiness, and the lesions may appear red-pink on lighter skin and gray-brown on darker skin types. Although these infections can arise in a variety of combinations, two feet–one hand syndrome occurs in about 60% of cases. <em>Trichophyton rubrum</em> is the most common agent. <br/><br/>[[{"fid":"293565","view_mode":"medstat_image_flush_left","fields":{"format":"medstat_image_flush_left","field_file_image_alt_text[und][0][value]":"","field_file_image_credit[und][0][value]":"Dr. Donna Bilu Martin","field_file_image_caption[und][0][value]":""},"type":"media","attributes":{"class":"media-element file-medstat_image_flush_left"}}]]Diagnosis is made by patient history, dermoscopic visualization, and staining of skin scraping with KOH or fungal culture. Dermatophytes prefer moist, warm environments, so this disease is prevalent in tropical conditions and associated with moist public areas such as locker rooms and showers. As a result, tinea pedis is also nicknamed “athlete’s foot” for its common presentation in athletes. The fungus spreads easily through contact and can survive on infected surfaces, so patients often self-inoculate by touching/scratching the affected area then touching another body part. Cautions that should be taken to avoid transmission include not sharing personal care products, washing the area and keeping it dry, and avoiding close, humid environments.<br/><br/>[[{"fid":"293566","view_mode":"medstat_image_flush_right","fields":{"format":"medstat_image_flush_right","field_file_image_alt_text[und][0][value]":"","field_file_image_credit[und][0][value]":"Dr. Donna Bilu Martin","field_file_image_caption[und][0][value]":""},"type":"media","attributes":{"class":"media-element file-medstat_image_flush_right"}}]]The syndrome is highly associated with onychomycosis, which can be more difficult to treat and often requires oral antifungals. Tinea manuum is commonly misdiagnosed as hand dermatitis or eczema and treated with topical steroids, which will exacerbate or flare the tinea. <br/><br/>Two feet–one hand syndrome can typically be treated with over-the-counter topical antifungal medications such as miconazole or clotrimazole. Topical ketoconazole may be prescribed, and oral terbinafine or itraconazole are used in more severe cases when a larger body surface area is affected or in immunocompromised patients. <br/><br/>[[{"fid":"293567","view_mode":"medstat_image_flush_right","fields":{"format":"medstat_image_flush_right","field_file_image_alt_text[und][0][value]":"","field_file_image_credit[und][0][value]":"Dr. Donna Bilu Martin","field_file_image_caption[und][0][value]":""},"type":"media","attributes":{"class":"media-element file-medstat_image_flush_right"}}]]This case and photo were submitted by Lucas Shapiro, BS, Nova Southeastern University, Davie, Fla.; Kiran C. Patel, Tampa Bay Regional Campus; and Dr. Bilu Martin.<br/><br/></p> <p> <em>Dr. Bilu Martin is a board-certified dermatologist in private practice at Premier Dermatology, MD, in Aventura, Fla. More diagnostic cases are available at mdedge.com/dermatology. To submit a case for possible publication, send an email to dermnews@mdedge.com. </em> </p> <p> [[{"fid":"271802","view_mode":"medstat_image_flush_right","fields":{"format":"medstat_image_flush_right","field_file_image_alt_text[und][0][value]":"Dr. Donna Bilu Martin, Premier Dermatology, MD, Aventura, Fla.","field_file_image_credit[und][0][value]":"","field_file_image_caption[und][0][value]":"Dr. Donna Bilu Martin"},"type":"media","attributes":{"class":"media-element file-medstat_image_flush_right"}}]] </p> <h2>References</h2> <p>Cleveland Clinic. <a href="https://my.clevelandclinic.org/health/diseases/24063-tinea-manuum">Tinea manuum: Symptoms, causes &amp; treatment</a>. 2022. https://my.clevelandclinic.org/health/diseases/24063-tinea-manuum. <br/><br/>Ugalde-Trejo NX et al. <a href="https://link.springer.com/article/10.1007/s12281-022-00447-9#article-info">Curr Fungal Infect Rep. 2022 Nov 17. doi: 10.1007/s12281-022-00447-9.</a><br/><br/>Mizumoto J. <a href="https://www.ncbi.nlm.nih.gov/pmc/articles/PMC8791668/">Cureus. 2021 Dec 27;13(12):e20758</a>.</p> </itemContent> </newsItem> <newsItem> <itemMeta> <itemRole>teaser</itemRole> <itemClass>text</itemClass> <title/> <deck/> </itemMeta> <itemContent> </itemContent> </newsItem> </itemSet></root>
Questionnaire Body

162413_foot_web.jpg
A 47-year-old White male with no significant medical history presented with pruritic, scaly, erythematous patches on the feet and left hand. 

Disallow All Ads
Content Gating
No Gating (article Unlocked/Free)
Alternative CME
Disqus Comments
Default
Use ProPublica
Hide sidebar & use full width
render the right sidebar.
Conference Recap Checkbox
Not Conference Recap
Clinical Edge
Display the Slideshow in this Article
Medscape Article
Display survey writer
Reuters content
Disable Inline Native ads
WebMD Article

A 50-year-old woman with no significant history presented with erythematous, annular plaques, and papules on the dorsal hands and arms

Article Type
Changed
Fri, 01/13/2023 - 12:37

Granuloma annulare (GA) is a self-limiting condition, and is known as the most common noninfectious granulomatous disease. The prevalence and incidence is approximately 0.1%-0.4%. Although the condition is benign, it may be associated with more serious conditions such as HIV and malignancy. GA affects women more frequently than men but can affect any age group, although it most commonly presents in those ages 30 years and younger. While the exact etiology is unknown, GA has been most strongly associated with diabetes mellitus, hyperlipidemia, and autoimmune diseases.

161783_left_hand_web.jpg

The disease presents as localized, annular erythematous plaques and papules on the dorsal hands and feet in approximately 75% of cases. However, eruptions may appear on the trunk and extremities and can be categorized into patchy, generalized, interstitial, subcutaneous, or perforating subtypes. The lesions are often asymptomatic and typically not associated with any other symptoms.

161783_arm_web.jpg

The pathogenesis of GA is still under investigation, but recent studies suggest that a Th1-mediated dysregulation of the JAK-STAT pathway may contribute to the disease. Other hypotheses include a delayed hypersensitivity reaction or cell mediated immune response. The mechanism may be multifaceted, and epidemiologic research suggests a genetic predisposition in White individuals, but these findings may be associated with socioeconomic factors and disparities in health care.

GA presents on histology with palisading histiocytes surrounding focal collagen necrobiosis with mucin deposition. Tissue samples also display leukocytic infiltration of the dermis featuring multinucleated giant cells. There are defining features of the different subtypes, but focal collagen necrosis, the presence of histiocytes, and mucin deposition are consistent findings across all presentations.

GA lesions commonly regress on their own, but they tend to recur and can be functionally and visually unappealing to patients. The most common treatments for GA include topical corticosteroids, intralesional corticosteroid injections, and other anti-inflammatory drugs. These interventions can be administered in a variety of ways as the inflammation caused by GA exists on a spectrum, and less severe cases can be managed with topical or intralesional treatment. Systemic therapy may be necessary for severe and recalcitrant cases. Other interventions that have shown promise in smaller studies include phototherapy, hydroxychloroquine, and TNF-alpha inhibitors.

BiluMartin_Donna_FLORIDA_web.jpg
Dr. Donna Bilu Martin


This case and photo were submitted by Lucas Shapiro, BS, Nova Southeastern University College of Osteopathic Medicine, Tampa Bay Regional Campus, and Dr. Bilu Martin.
 

Dr. Bilu Martin is a board-certified dermatologist in private practice at Premier Dermatology, MD, in Aventura, Fla. More diagnostic cases are available at mdedge.com/dermatology. To submit a case for possible publication, send an email to dermnews@mdedge.com.

References

Joshi TP and Duvic M. Am J Clin Dermatol. 2022 Jan;23(1):37-50. doi: 10.1007/s40257-021-00636-1.

Muse M et al. Dermatol Online J. 2021 Apr 15;27(4):13030/qt0m50398n.

Schmieder SJ et al. Granuloma Annulare. NIH National Center for Biotechnology Information [Updated 2022 Nov 7]. In: StatPearls [Internet]. Treasure Island (FL): StatPearls Publishing; 2022 Jan. 7.

Publications
Topics
Sections

Granuloma annulare (GA) is a self-limiting condition, and is known as the most common noninfectious granulomatous disease. The prevalence and incidence is approximately 0.1%-0.4%. Although the condition is benign, it may be associated with more serious conditions such as HIV and malignancy. GA affects women more frequently than men but can affect any age group, although it most commonly presents in those ages 30 years and younger. While the exact etiology is unknown, GA has been most strongly associated with diabetes mellitus, hyperlipidemia, and autoimmune diseases.

161783_left_hand_web.jpg

The disease presents as localized, annular erythematous plaques and papules on the dorsal hands and feet in approximately 75% of cases. However, eruptions may appear on the trunk and extremities and can be categorized into patchy, generalized, interstitial, subcutaneous, or perforating subtypes. The lesions are often asymptomatic and typically not associated with any other symptoms.

161783_arm_web.jpg

The pathogenesis of GA is still under investigation, but recent studies suggest that a Th1-mediated dysregulation of the JAK-STAT pathway may contribute to the disease. Other hypotheses include a delayed hypersensitivity reaction or cell mediated immune response. The mechanism may be multifaceted, and epidemiologic research suggests a genetic predisposition in White individuals, but these findings may be associated with socioeconomic factors and disparities in health care.

GA presents on histology with palisading histiocytes surrounding focal collagen necrobiosis with mucin deposition. Tissue samples also display leukocytic infiltration of the dermis featuring multinucleated giant cells. There are defining features of the different subtypes, but focal collagen necrosis, the presence of histiocytes, and mucin deposition are consistent findings across all presentations.

GA lesions commonly regress on their own, but they tend to recur and can be functionally and visually unappealing to patients. The most common treatments for GA include topical corticosteroids, intralesional corticosteroid injections, and other anti-inflammatory drugs. These interventions can be administered in a variety of ways as the inflammation caused by GA exists on a spectrum, and less severe cases can be managed with topical or intralesional treatment. Systemic therapy may be necessary for severe and recalcitrant cases. Other interventions that have shown promise in smaller studies include phototherapy, hydroxychloroquine, and TNF-alpha inhibitors.

BiluMartin_Donna_FLORIDA_web.jpg
Dr. Donna Bilu Martin


This case and photo were submitted by Lucas Shapiro, BS, Nova Southeastern University College of Osteopathic Medicine, Tampa Bay Regional Campus, and Dr. Bilu Martin.
 

Dr. Bilu Martin is a board-certified dermatologist in private practice at Premier Dermatology, MD, in Aventura, Fla. More diagnostic cases are available at mdedge.com/dermatology. To submit a case for possible publication, send an email to dermnews@mdedge.com.

References

Joshi TP and Duvic M. Am J Clin Dermatol. 2022 Jan;23(1):37-50. doi: 10.1007/s40257-021-00636-1.

Muse M et al. Dermatol Online J. 2021 Apr 15;27(4):13030/qt0m50398n.

Schmieder SJ et al. Granuloma Annulare. NIH National Center for Biotechnology Information [Updated 2022 Nov 7]. In: StatPearls [Internet]. Treasure Island (FL): StatPearls Publishing; 2022 Jan. 7.

Granuloma annulare (GA) is a self-limiting condition, and is known as the most common noninfectious granulomatous disease. The prevalence and incidence is approximately 0.1%-0.4%. Although the condition is benign, it may be associated with more serious conditions such as HIV and malignancy. GA affects women more frequently than men but can affect any age group, although it most commonly presents in those ages 30 years and younger. While the exact etiology is unknown, GA has been most strongly associated with diabetes mellitus, hyperlipidemia, and autoimmune diseases.

161783_left_hand_web.jpg

The disease presents as localized, annular erythematous plaques and papules on the dorsal hands and feet in approximately 75% of cases. However, eruptions may appear on the trunk and extremities and can be categorized into patchy, generalized, interstitial, subcutaneous, or perforating subtypes. The lesions are often asymptomatic and typically not associated with any other symptoms.

161783_arm_web.jpg

The pathogenesis of GA is still under investigation, but recent studies suggest that a Th1-mediated dysregulation of the JAK-STAT pathway may contribute to the disease. Other hypotheses include a delayed hypersensitivity reaction or cell mediated immune response. The mechanism may be multifaceted, and epidemiologic research suggests a genetic predisposition in White individuals, but these findings may be associated with socioeconomic factors and disparities in health care.

GA presents on histology with palisading histiocytes surrounding focal collagen necrobiosis with mucin deposition. Tissue samples also display leukocytic infiltration of the dermis featuring multinucleated giant cells. There are defining features of the different subtypes, but focal collagen necrosis, the presence of histiocytes, and mucin deposition are consistent findings across all presentations.

GA lesions commonly regress on their own, but they tend to recur and can be functionally and visually unappealing to patients. The most common treatments for GA include topical corticosteroids, intralesional corticosteroid injections, and other anti-inflammatory drugs. These interventions can be administered in a variety of ways as the inflammation caused by GA exists on a spectrum, and less severe cases can be managed with topical or intralesional treatment. Systemic therapy may be necessary for severe and recalcitrant cases. Other interventions that have shown promise in smaller studies include phototherapy, hydroxychloroquine, and TNF-alpha inhibitors.

BiluMartin_Donna_FLORIDA_web.jpg
Dr. Donna Bilu Martin


This case and photo were submitted by Lucas Shapiro, BS, Nova Southeastern University College of Osteopathic Medicine, Tampa Bay Regional Campus, and Dr. Bilu Martin.
 

Dr. Bilu Martin is a board-certified dermatologist in private practice at Premier Dermatology, MD, in Aventura, Fla. More diagnostic cases are available at mdedge.com/dermatology. To submit a case for possible publication, send an email to dermnews@mdedge.com.

References

Joshi TP and Duvic M. Am J Clin Dermatol. 2022 Jan;23(1):37-50. doi: 10.1007/s40257-021-00636-1.

Muse M et al. Dermatol Online J. 2021 Apr 15;27(4):13030/qt0m50398n.

Schmieder SJ et al. Granuloma Annulare. NIH National Center for Biotechnology Information [Updated 2022 Nov 7]. In: StatPearls [Internet]. Treasure Island (FL): StatPearls Publishing; 2022 Jan. 7.

Publications
Publications
Topics
Article Type
Sections
Teambase XML
<?xml version="1.0" encoding="UTF-8"?>
<!--$RCSfile: InCopy_agile.xsl,v $ $Revision: 1.35 $-->
<!--$RCSfile: drupal.xsl,v $ $Revision: 1.7 $-->
<root generator="drupal.xsl" gversion="1.7"> <header> <fileName>161783</fileName> <TBEID>0C047B08.SIG</TBEID> <TBUniqueIdentifier>MD_0C047B08</TBUniqueIdentifier> <newsOrJournal>News</newsOrJournal> <publisherName>Frontline Medical Communications</publisherName> <storyname>Feb make the Dx</storyname> <articleType>353</articleType> <TBLocation>QC Done-All Pubs</TBLocation> <QCDate>20230113T115317</QCDate> <firstPublished>20230113T122742</firstPublished> <LastPublished>20230113T122742</LastPublished> <pubStatus qcode="stat:"/> <embargoDate/> <killDate/> <CMSDate>20230113T122742</CMSDate> <articleSource/> <facebookInfo/> <meetingNumber/> <byline>Bilu Martin and Shapiro</byline> <bylineText>LUCAS SHAPIRO AND DONNA BILU MARTIN, MD</bylineText> <bylineFull>LUCAS SHAPIRO AND DONNA BILU MARTIN, MD</bylineFull> <bylineTitleText/> <USOrGlobal/> <wireDocType/> <newsDocType>Column</newsDocType> <journalDocType/> <linkLabel/> <pageRange/> <citation/> <quizID/> <indexIssueDate/> <itemClass qcode="ninat:text"/> <provider qcode="provider:imng"> <name>IMNG Medical Media</name> <rightsInfo> <copyrightHolder> <name>Frontline Medical News</name> </copyrightHolder> <copyrightNotice>Copyright (c) 2015 Frontline Medical News, a Frontline Medical Communications Inc. company. All rights reserved. This material may not be published, broadcast, copied, or otherwise reproduced or distributed without the prior written permission of Frontline Medical Communications Inc.</copyrightNotice> </rightsInfo> </provider> <abstract/> <metaDescription>Granuloma annulare (GA) is a self-limiting condition, and is known as the most common noninfectious granulomatous disease</metaDescription> <articlePDF/> <teaserImage>292308</teaserImage> <teaser>GA presents on histology with palisading histiocytes surrounding focal collagen necrobiosis with mucin deposition.</teaser> <title>Granuloma annulare</title> <deck/> <disclaimer/> <AuthorList/> <articleURL/> <doi/> <pubMedID/> <publishXMLStatus/> <publishXMLVersion>1</publishXMLVersion> <useEISSN>0</useEISSN> <urgency/> <pubPubdateYear/> <pubPubdateMonth/> <pubPubdateDay/> <pubVolume/> <pubNumber/> <wireChannels/> <primaryCMSID/> <CMSIDs/> <keywords/> <seeAlsos/> <publications_g> <publicationData> <publicationCode>skin</publicationCode> <pubIssueName/> <pubArticleType/> <pubTopics/> <pubCategories/> <pubSections/> </publicationData> <publicationData> <publicationCode>fp</publicationCode> <pubIssueName/> <pubArticleType/> <pubTopics/> <pubCategories/> <pubSections/> </publicationData> <publicationData> <publicationCode>im</publicationCode> <pubIssueName/> <pubArticleType/> <pubTopics/> <pubCategories/> <pubSections/> </publicationData> </publications_g> <publications> <term canonical="true">13</term> <term>15</term> <term>21</term> </publications> <sections> <term canonical="true">87</term> <term>52</term> </sections> <topics> <term canonical="true">39212</term> <term>203</term> </topics> <links> <link> <itemClass qcode="ninat:picture"/> <altRep contenttype="image/jpeg">images/240117de.jpg</altRep> <description role="drol:caption"/> <description role="drol:credit">Courtesy Lucas Shapiro and Dr. Bilu Martin</description> </link> <link> <itemClass qcode="ninat:picture"/> <altRep contenttype="image/jpeg">images/240117dd.jpg</altRep> <description role="drol:caption"/> <description role="drol:credit">Courtesy Lucas Shapiro and Dr. Bilu Martin</description> </link> <link> <itemClass qcode="ninat:picture"/> <altRep contenttype="image/jpeg">images/240117dc.jpg</altRep> <description role="drol:caption"/> <description role="drol:credit">Courtesy Lucas Shapiro and Dr. Bilu Martin</description> </link> <link> <itemClass qcode="ninat:picture"/> <altRep contenttype="image/jpeg">images/2400f1a6.jpg</altRep> <description role="drol:caption">Dr. Donna Bilu Martin</description> <description role="drol:credit"/> </link> </links> </header> <itemSet> <newsItem> <itemMeta> <itemRole>Main</itemRole> <itemClass>text</itemClass> <title>Granuloma annulare</title> <deck/> </itemMeta> <itemContent> <p><span class="tag metaDescription">Granuloma annulare (GA) is a self-limiting condition, and is known as the most common noninfectious granulomatous disease</span>. The prevalence and incidence is approximately 0.1%-0.4%. Although the condition is benign, it may be associated with more serious conditions such as HIV and malignancy. GA affects women more frequently than men but can affect any age group, although it most commonly presents in those ages 30 years and younger. While the exact etiology is unknown, GA has been most strongly associated with diabetes mellitus, hyperlipidemia, and autoimmune diseases.</p> <p>[[{"fid":"292308","view_mode":"medstat_image_flush_left","fields":{"format":"medstat_image_flush_left","field_file_image_alt_text[und][0][value]":"","field_file_image_credit[und][0][value]":"Courtesy Lucas Shapiro and Dr. Bilu Martin","field_file_image_caption[und][0][value]":""},"type":"media","attributes":{"class":"media-element file-medstat_image_flush_left"}}]]The disease presents as localized, annular erythematous plaques and papules on the dorsal hands and feet in approximately 75% of cases. However, eruptions may appear on the trunk and extremities and can be categorized into patchy, generalized, interstitial, subcutaneous, or perforating subtypes. The lesions are often asymptomatic and typically not associated with any other symptoms.<br/><br/>[[{"fid":"292307","view_mode":"medstat_image_flush_left","fields":{"format":"medstat_image_flush_left","field_file_image_alt_text[und][0][value]":"","field_file_image_credit[und][0][value]":"Courtesy Lucas Shapiro and Dr. Bilu Martin","field_file_image_caption[und][0][value]":""},"type":"media","attributes":{"class":"media-element file-medstat_image_flush_left"}}]]The pathogenesis of GA is still under investigation, but recent studies suggest that a Th1-mediated dysregulation of the JAK-STAT pathway may contribute to the disease. Other hypotheses include a delayed hypersensitivity reaction or cell mediated immune response. The mechanism may be multifaceted, and epidemiologic research suggests a genetic predisposition in White individuals, but these findings may be associated with socioeconomic factors and disparities in health care.<br/><br/>[[{"fid":"292306","view_mode":"medstat_image_flush_right","fields":{"format":"medstat_image_flush_right","field_file_image_alt_text[und][0][value]":"","field_file_image_credit[und][0][value]":"Courtesy Lucas Shapiro and Dr. Bilu Martin","field_file_image_caption[und][0][value]":""},"type":"media","attributes":{"class":"media-element file-medstat_image_flush_right"}}]]GA presents on histology with palisading histiocytes surrounding focal collagen necrobiosis with mucin deposition. Tissue samples also display leukocytic infiltration of the dermis featuring multinucleated giant cells. There are defining features of the different subtypes, but focal collagen necrosis, the presence of histiocytes, and mucin deposition are consistent findings across all presentations. <br/><br/>GA lesions commonly regress on their own, but they tend to recur and can be functionally and visually unappealing to patients. The most common treatments for GA include topical corticosteroids, intralesional corticosteroid injections, and other anti-inflammatory drugs. These interventions can be administered in a variety of ways as the inflammation caused by GA exists on a spectrum, and less severe cases can be managed with topical or intralesional treatment. Systemic therapy may be necessary for severe and recalcitrant cases. Other interventions that have shown promise in smaller studies include phototherapy, hydroxychloroquine, and TNF-alpha inhibitors. [[{"fid":"271802","view_mode":"medstat_image_flush_right","fields":{"format":"medstat_image_flush_right","field_file_image_alt_text[und][0][value]":"Dr. Donna Bilu Martin, Premier Dermatology, MD, Aventura, Fla.","field_file_image_credit[und][0][value]":"","field_file_image_caption[und][0][value]":"Dr. Donna Bilu Martin"},"type":"media","attributes":{"class":"media-element file-medstat_image_flush_right"}}]]<br/><br/>This case and photo were submitted by Lucas Shapiro, BS, Nova Southeastern University College of Osteopathic Medicine, Tampa Bay Regional Campus, and Dr. Bilu Martin.<br/><br/></p> <p> <em>Dr. Bilu Martin is a board-certified dermatologist in private practice at Premier Dermatology, MD, in Aventura, Fla. More diagnostic cases are available at <span class="Hyperlink"><a href="http://mdedge.com/dermatology">mdedge.com/dermatology</a></span>. To submit a case for possible publication, send an email to <span class="Hyperlink"><a href="mailto:dermnews%40mdedge.com?subject=">dermnews@mdedge.com</a></span>.</em> </p> <h2>References</h2> <p>Joshi TP and Duvic M. <span class="Hyperlink"><a href="https://www.ncbi.nlm.nih.gov/pmc/articles/PMC8423598/">Am J Clin Dermatol. 2022 Jan;23(1):37-50. doi: 10.1007/s40257-021-00636-1</a></span>.<br/><br/>Muse M et al. <span class="Hyperlink"><a href="https://escholarship.org/uc/item/0m50398n">Dermatol Online J. 2021 Apr 15;27(4):13030/qt0m50398n</a></span>.<br/><br/>Schmieder SJ et al. <span class="Hyperlink"><a href="https://www.ncbi.nlm.nih.gov/books/NBK459377/">Granuloma Annulare</a></span>. NIH National Center for Biotechnology Information [Updated 2022 Nov 7]. In: StatPearls [Internet]. Treasure Island (FL): StatPearls Publishing; 2022 Jan. 7.</p> </itemContent> </newsItem> <newsItem> <itemMeta> <itemRole>teaser</itemRole> <itemClass>text</itemClass> <title/> <deck/> </itemMeta> <itemContent> </itemContent> </newsItem> </itemSet></root>
Questionnaire Body

161783_both_hands_web.jpg
A 50-year-old woman with no significant medical history presented with erythematous, annular plaques and papules on the dorsal hands and arms. The lesions have been present for years, and have previously been treated with topical and intralesional steroids. 

Disallow All Ads
Content Gating
No Gating (article Unlocked/Free)
Alternative CME
Disqus Comments
Default
Use ProPublica
Hide sidebar & use full width
render the right sidebar.
Conference Recap Checkbox
Not Conference Recap
Clinical Edge
Display the Slideshow in this Article
Medscape Article
Display survey writer
Reuters content
Disable Inline Native ads
WebMD Article

​​​​​​​A 9-year old female presented with 1 day of fever, fatigue, and sore throat

Article Type
Changed
Wed, 12/07/2022 - 16:22

Scarlet fever, commonly described in young children and adolescents, is characterized by a papular, blanching rash that may be described as having a “sandpaper” texture. This condition typically presents in the setting of Streptococcus pyogenes pharyngitis, or strep throat, and is spread via mucosal transfer in close proximity such as classrooms and nurseries. The dermatologic symptoms are a result of the endotoxin produced by S. pyogenes, which is part of the group A Strep bacteria. Clinically, the presentation can be differentiated from an allergic eruption by its relation to acute pharyngitis, insidious onset, and lack of confluence of the lesions. Diagnosis is supported by a throat culture and rapid strep test, although a rapid test lacks reliability in older patients who are less commonly affected and likely to be carriers. First-line treatment is penicillin or amoxicillin, but first-generation cephalosporins, clindamycin, or erythromycin are sufficient if the patient is allergic to penicillins. Prognosis worsens as time between onset and treatment increases, but is overall excellent now with the introduction of antibiotics and improved hygiene.

Scarlet fever is among a list of many common childhood rashes, and it can be difficult to differentiate between these pathologies on clinical presentation. A few notable childhood dermatologic eruptions include erythema infectiosum (fifth disease), roseola (exanthema subitum or sixth disease), and measles. These cases can be distinguished clinically by the age of the patient, distribution, and quality of the symptoms. Laboratory testing may be used to confirm the diagnosis.

BiluMartin_Donna_FLORIDA_web.jpg
Dr. Donna Bilu Martin

Erythema infectiosum is known as fifth disease or slapped-cheek rash because it commonly presents on the cheeks as a pink, maculopapular rash in a reticular pattern. The disease is caused by parvovirus B19 and is accompanied by low fever, malaise, headache, sore throat, and nausea, which precedes the erythematous rash. The facial rash appears first and is followed by patchy eruptions on the extremities. Appearance of the rash typically indicates the patient is no longer contagious, and patients are treated symptomatically with NSAIDs and antihistamines for associated pruritus.

Roseola infantum is commonly caused by human herpesvirus 6 and is usually found in children 3 years and younger. The defining symptom is a high fever, which is paired with a mild cough, runny nose, and diarrhea. A maculopapular rash appears after the fever subsides, starting centrally and spreading outward to the extremities. Although this rash is similar to measles, they can be differentiated by the order of onset. The rash caused by measles begins on the face and mouth (Koplik spots) and moves downward. Additionally, the patient appears generally healthy and the disease is self-limiting in roseola, while patients with measles will appear more ill and require further attention. Measles is caused by the measles virus of the genus Morbillivirus and is highly contagious. It is spread via respiratory route presenting with fever, cough, coryza, and conjunctivitis followed by the rash. Fortunately, the measles vaccine is in widespread use, so cases have declined over the years.

Our patient had a positive strep test. Influenza and coronavirus tests were negative. She was started on daily amoxicillin and the rash resolved within 2 days of taking the antibiotics.

This case and photo were submitted by Lucas Shapiro, BS, Nova Southeastern University, Tampa, and Dr. Bilu Martin.

Dr. Bilu Martin is a board-certified dermatologist in private practice at Premier Dermatology, MD, in Aventura, Fla. More diagnostic cases are available at mdedge.com/dermatology. To submit a case for possible publication, send an email to dermnews@mdedge.com.

References

Allmon A et al.. Am Fam Physician. 2015 Aug 1;92(3):211-6.

Moss WJ. Lancet. 2017 Dec 2;390(10111):2490-502.

Mullins TB and Krishnamurthy K. Roseola Infantum, in “StatPearls.” Treasure Islan, Fla.: StatPearls Publishing, 2022.

Pardo S and Perera TB. Scarlet Fever, in “StatPearls.” Treasure Island, Fla.: StatPearls Publishing, 2022.
 

Publications
Topics
Sections

Scarlet fever, commonly described in young children and adolescents, is characterized by a papular, blanching rash that may be described as having a “sandpaper” texture. This condition typically presents in the setting of Streptococcus pyogenes pharyngitis, or strep throat, and is spread via mucosal transfer in close proximity such as classrooms and nurseries. The dermatologic symptoms are a result of the endotoxin produced by S. pyogenes, which is part of the group A Strep bacteria. Clinically, the presentation can be differentiated from an allergic eruption by its relation to acute pharyngitis, insidious onset, and lack of confluence of the lesions. Diagnosis is supported by a throat culture and rapid strep test, although a rapid test lacks reliability in older patients who are less commonly affected and likely to be carriers. First-line treatment is penicillin or amoxicillin, but first-generation cephalosporins, clindamycin, or erythromycin are sufficient if the patient is allergic to penicillins. Prognosis worsens as time between onset and treatment increases, but is overall excellent now with the introduction of antibiotics and improved hygiene.

Scarlet fever is among a list of many common childhood rashes, and it can be difficult to differentiate between these pathologies on clinical presentation. A few notable childhood dermatologic eruptions include erythema infectiosum (fifth disease), roseola (exanthema subitum or sixth disease), and measles. These cases can be distinguished clinically by the age of the patient, distribution, and quality of the symptoms. Laboratory testing may be used to confirm the diagnosis.

BiluMartin_Donna_FLORIDA_web.jpg
Dr. Donna Bilu Martin

Erythema infectiosum is known as fifth disease or slapped-cheek rash because it commonly presents on the cheeks as a pink, maculopapular rash in a reticular pattern. The disease is caused by parvovirus B19 and is accompanied by low fever, malaise, headache, sore throat, and nausea, which precedes the erythematous rash. The facial rash appears first and is followed by patchy eruptions on the extremities. Appearance of the rash typically indicates the patient is no longer contagious, and patients are treated symptomatically with NSAIDs and antihistamines for associated pruritus.

Roseola infantum is commonly caused by human herpesvirus 6 and is usually found in children 3 years and younger. The defining symptom is a high fever, which is paired with a mild cough, runny nose, and diarrhea. A maculopapular rash appears after the fever subsides, starting centrally and spreading outward to the extremities. Although this rash is similar to measles, they can be differentiated by the order of onset. The rash caused by measles begins on the face and mouth (Koplik spots) and moves downward. Additionally, the patient appears generally healthy and the disease is self-limiting in roseola, while patients with measles will appear more ill and require further attention. Measles is caused by the measles virus of the genus Morbillivirus and is highly contagious. It is spread via respiratory route presenting with fever, cough, coryza, and conjunctivitis followed by the rash. Fortunately, the measles vaccine is in widespread use, so cases have declined over the years.

Our patient had a positive strep test. Influenza and coronavirus tests were negative. She was started on daily amoxicillin and the rash resolved within 2 days of taking the antibiotics.

This case and photo were submitted by Lucas Shapiro, BS, Nova Southeastern University, Tampa, and Dr. Bilu Martin.

Dr. Bilu Martin is a board-certified dermatologist in private practice at Premier Dermatology, MD, in Aventura, Fla. More diagnostic cases are available at mdedge.com/dermatology. To submit a case for possible publication, send an email to dermnews@mdedge.com.

References

Allmon A et al.. Am Fam Physician. 2015 Aug 1;92(3):211-6.

Moss WJ. Lancet. 2017 Dec 2;390(10111):2490-502.

Mullins TB and Krishnamurthy K. Roseola Infantum, in “StatPearls.” Treasure Islan, Fla.: StatPearls Publishing, 2022.

Pardo S and Perera TB. Scarlet Fever, in “StatPearls.” Treasure Island, Fla.: StatPearls Publishing, 2022.
 

Scarlet fever, commonly described in young children and adolescents, is characterized by a papular, blanching rash that may be described as having a “sandpaper” texture. This condition typically presents in the setting of Streptococcus pyogenes pharyngitis, or strep throat, and is spread via mucosal transfer in close proximity such as classrooms and nurseries. The dermatologic symptoms are a result of the endotoxin produced by S. pyogenes, which is part of the group A Strep bacteria. Clinically, the presentation can be differentiated from an allergic eruption by its relation to acute pharyngitis, insidious onset, and lack of confluence of the lesions. Diagnosis is supported by a throat culture and rapid strep test, although a rapid test lacks reliability in older patients who are less commonly affected and likely to be carriers. First-line treatment is penicillin or amoxicillin, but first-generation cephalosporins, clindamycin, or erythromycin are sufficient if the patient is allergic to penicillins. Prognosis worsens as time between onset and treatment increases, but is overall excellent now with the introduction of antibiotics and improved hygiene.

Scarlet fever is among a list of many common childhood rashes, and it can be difficult to differentiate between these pathologies on clinical presentation. A few notable childhood dermatologic eruptions include erythema infectiosum (fifth disease), roseola (exanthema subitum or sixth disease), and measles. These cases can be distinguished clinically by the age of the patient, distribution, and quality of the symptoms. Laboratory testing may be used to confirm the diagnosis.

BiluMartin_Donna_FLORIDA_web.jpg
Dr. Donna Bilu Martin

Erythema infectiosum is known as fifth disease or slapped-cheek rash because it commonly presents on the cheeks as a pink, maculopapular rash in a reticular pattern. The disease is caused by parvovirus B19 and is accompanied by low fever, malaise, headache, sore throat, and nausea, which precedes the erythematous rash. The facial rash appears first and is followed by patchy eruptions on the extremities. Appearance of the rash typically indicates the patient is no longer contagious, and patients are treated symptomatically with NSAIDs and antihistamines for associated pruritus.

Roseola infantum is commonly caused by human herpesvirus 6 and is usually found in children 3 years and younger. The defining symptom is a high fever, which is paired with a mild cough, runny nose, and diarrhea. A maculopapular rash appears after the fever subsides, starting centrally and spreading outward to the extremities. Although this rash is similar to measles, they can be differentiated by the order of onset. The rash caused by measles begins on the face and mouth (Koplik spots) and moves downward. Additionally, the patient appears generally healthy and the disease is self-limiting in roseola, while patients with measles will appear more ill and require further attention. Measles is caused by the measles virus of the genus Morbillivirus and is highly contagious. It is spread via respiratory route presenting with fever, cough, coryza, and conjunctivitis followed by the rash. Fortunately, the measles vaccine is in widespread use, so cases have declined over the years.

Our patient had a positive strep test. Influenza and coronavirus tests were negative. She was started on daily amoxicillin and the rash resolved within 2 days of taking the antibiotics.

This case and photo were submitted by Lucas Shapiro, BS, Nova Southeastern University, Tampa, and Dr. Bilu Martin.

Dr. Bilu Martin is a board-certified dermatologist in private practice at Premier Dermatology, MD, in Aventura, Fla. More diagnostic cases are available at mdedge.com/dermatology. To submit a case for possible publication, send an email to dermnews@mdedge.com.

References

Allmon A et al.. Am Fam Physician. 2015 Aug 1;92(3):211-6.

Moss WJ. Lancet. 2017 Dec 2;390(10111):2490-502.

Mullins TB and Krishnamurthy K. Roseola Infantum, in “StatPearls.” Treasure Islan, Fla.: StatPearls Publishing, 2022.

Pardo S and Perera TB. Scarlet Fever, in “StatPearls.” Treasure Island, Fla.: StatPearls Publishing, 2022.
 

Publications
Publications
Topics
Article Type
Sections
Teambase XML
<?xml version="1.0" encoding="UTF-8"?>
<!--$RCSfile: InCopy_agile.xsl,v $ $Revision: 1.35 $-->
<!--$RCSfile: drupal.xsl,v $ $Revision: 1.7 $-->
<root generator="drupal.xsl" gversion="1.7"> <header> <fileName>161245</fileName> <TBEID>0C046F00.SIG</TBEID> <TBUniqueIdentifier>MD_0C046F00</TBUniqueIdentifier> <newsOrJournal>News</newsOrJournal> <publisherName>Frontline Medical Communications</publisherName> <storyname>Jan. Make the Dx</storyname> <articleType>353</articleType> <TBLocation>QC Done-All Pubs</TBLocation> <QCDate>20221207T113851</QCDate> <firstPublished>20221207T161443</firstPublished> <LastPublished>20221207T161443</LastPublished> <pubStatus qcode="stat:"/> <embargoDate/> <killDate/> <CMSDate>20221207T161443</CMSDate> <articleSource/> <facebookInfo/> <meetingNumber/> <byline>Shapiro and Bilu Martin</byline> <bylineText>LUCAS SHAPIRO AND DONNA BILU MARTIN, MD</bylineText> <bylineFull>LUCAS SHAPIRO AND DONNA BILU MARTIN, MD</bylineFull> <bylineTitleText/> <USOrGlobal/> <wireDocType/> <newsDocType>Column</newsDocType> <journalDocType/> <linkLabel/> <pageRange/> <citation/> <quizID/> <indexIssueDate/> <itemClass qcode="ninat:text"/> <provider qcode="provider:imng"> <name>IMNG Medical Media</name> <rightsInfo> <copyrightHolder> <name>Frontline Medical News</name> </copyrightHolder> <copyrightNotice>Copyright (c) 2015 Frontline Medical News, a Frontline Medical Communications Inc. company. All rights reserved. This material may not be published, broadcast, copied, or otherwise reproduced or distributed without the prior written permission of Frontline Medical Communications Inc.</copyrightNotice> </rightsInfo> </provider> <abstract/> <metaDescription>Scarlet fever, commonly described in young children and adolescents, is characterized by a papular, blanching rash that may be described as having a “sandpaper”</metaDescription> <articlePDF/> <teaserImage>271802</teaserImage> <teaser>Scarlet fever is among a list of many common childhood rashes, and it can be difficult to differentiate between these pathologies on clinical presentation.</teaser> <title>Scarlet fever</title> <deck/> <disclaimer/> <AuthorList/> <articleURL/> <doi/> <pubMedID/> <publishXMLStatus/> <publishXMLVersion>1</publishXMLVersion> <useEISSN>0</useEISSN> <urgency/> <pubPubdateYear/> <pubPubdateMonth/> <pubPubdateDay/> <pubVolume/> <pubNumber/> <wireChannels/> <primaryCMSID/> <CMSIDs/> <keywords/> <seeAlsos/> <publications_g> <publicationData> <publicationCode>skin</publicationCode> <pubIssueName/> <pubArticleType/> <pubTopics/> <pubCategories/> <pubSections/> </publicationData> <publicationData> <publicationCode>fp</publicationCode> <pubIssueName/> <pubArticleType/> <pubTopics/> <pubCategories/> <pubSections/> </publicationData> <publicationData> <publicationCode>pn</publicationCode> <pubIssueName/> <pubArticleType/> <pubTopics/> <pubCategories/> <pubSections/> </publicationData> </publications_g> <publications> <term canonical="true">13</term> <term>15</term> <term>25</term> </publications> <sections> <term canonical="true">87</term> <term>52</term> </sections> <topics> <term canonical="true">271</term> <term>234</term> <term>203</term> </topics> <links> <link> <itemClass qcode="ninat:picture"/> <altRep contenttype="image/jpeg">images/2400f1a6.jpg</altRep> <description role="drol:caption">Dr. Donna Bilu Martin</description> <description role="drol:credit"/> </link> </links> </header> <itemSet> <newsItem> <itemMeta> <itemRole>Main</itemRole> <itemClass>text</itemClass> <title>Scarlet fever</title> <deck/> </itemMeta> <itemContent> <p><span class="tag metaDescription">Scarlet fever, commonly described in young children and adolescents, is characterized by a papular, blanching rash that may be described as having a “sandpaper” texture.</span> This condition typically presents in the setting of <em>Streptococcus pyogenes</em> pharyngitis, or strep throat, and is spread via mucosal transfer in close proximity such as classrooms and nurseries. The dermatologic symptoms are a result of the endotoxin produced by <em>S. pyogenes</em>, which is part of the group A Strep bacteria. Clinically, the presentation can be differentiated from an allergic eruption by its relation to acute pharyngitis, insidious onset, and lack of confluence of the lesions. Diagnosis is supported by a throat culture and rapid strep test, although a rapid test lacks reliability in older patients who are less commonly affected and likely to be carriers. First-line treatment is penicillin or amoxicillin, but first-generation cephalosporins, clindamycin, or erythromycin are sufficient if the patient is allergic to penicillins. Prognosis worsens as time between onset and treatment increases, but is overall excellent now with the introduction of antibiotics and improved hygiene. </p> <p>Scarlet fever is among a list of many common childhood rashes, and it can be difficult to differentiate between these pathologies on clinical presentation. A few notable childhood dermatologic eruptions include erythema infectiosum (fifth disease), roseola (exanthema subitum or sixth disease), and measles. These cases can be distinguished clinically by the age of the patient, distribution, and quality of the symptoms. Laboratory testing may be used to confirm the diagnosis.<br/><br/>[[{"fid":"271802","view_mode":"medstat_image_flush_right","fields":{"format":"medstat_image_flush_right","field_file_image_alt_text[und][0][value]":"Dr. Donna Bilu Martin, Premier Dermatology, MD, Aventura, Fla.","field_file_image_credit[und][0][value]":"","field_file_image_caption[und][0][value]":"Dr. Donna Bilu Martin"},"type":"media","attributes":{"class":"media-element file-medstat_image_flush_right"}}]]Erythema infectiosum is known as fifth disease or slapped-cheek rash because it commonly presents on the cheeks as a pink, maculopapular rash in a reticular pattern. The disease is caused by parvovirus B19 and is accompanied by low fever, malaise, headache, sore throat, and nausea, which precedes the erythematous rash. The facial rash appears first and is followed by patchy eruptions on the extremities. Appearance of the rash typically indicates the patient is no longer contagious, and patients are treated symptomatically with NSAIDs and antihistamines for associated pruritus. <br/><br/>Roseola infantum is commonly caused by human herpesvirus 6 and is usually found in children 3 years and younger. The defining symptom is a high fever, which is paired with a mild cough, runny nose, and diarrhea. A maculopapular rash appears after the fever subsides, starting centrally and spreading outward to the extremities. Although this rash is similar to measles, they can be differentiated by the order of onset. The rash caused by measles begins on the face and mouth (Koplik spots) and moves downward. Additionally, the patient appears generally healthy and the disease is self-limiting in roseola, while patients with measles will appear more ill and require further attention. Measles is caused by the measles virus of the genus <em>Morbillivirus</em> and is highly contagious. It is spread via respiratory route presenting with fever, cough, coryza, and conjunctivitis followed by the rash. Fortunately, the measles vaccine is in widespread use, so cases have declined over the years. <br/><br/>Our patient had a positive strep test. Influenza and coronavirus tests were negative. She was started on daily amoxicillin and the rash resolved within 2 days of taking the antibiotics.<br/><br/>This case and photo were submitted by Lucas Shapiro, BS, Nova Southeastern University, Tampa, and Dr. Bilu Martin.</p> <p> <em>Dr. Bilu Martin is a board-certified dermatologist in private practice at Premier Dermatology, MD, in Aventura, Fla. More diagnostic cases are available at mdedge.com/dermatology. To submit a case for possible publication, send an email to dermnews@mdedge.com.</em> </p> <h2>References</h2> <p>Allmon A et al.. <span class="Hyperlink"><a href="https://pubmed.ncbi.nlm.nih.gov/26280141/">Am Fam Physician. 2015 Aug 1;92(3):211-6</a></span>. <br/><br/>Moss WJ. <span class="Hyperlink"><a href="https://www.thelancet.com/journals/lancet/article/PIIS0140-6736(17)31463-0/fulltext">Lancet. 2017 Dec 2;390(10111):2490-502</a></span>.<br/><br/>Mullins TB and Krishnamurthy K. <span class="Hyperlink"><a href="https://www.ncbi.nlm.nih.gov/books/NBK448190/">Roseola Infantum</a></span>, in “StatPearls.” Treasure Islan, Fla.: StatPearls Publishing, 2022. <br/><br/>Pardo S and Perera TB. <span class="Hyperlink"><a href="https://www.ncbi.nlm.nih.gov/books/NBK507889/">Scarlet Fever</a></span>, in “StatPearls.” Treasure Island, Fla.: StatPearls Publishing, 2022. <br/><br/></p> </itemContent> </newsItem> <newsItem> <itemMeta> <itemRole>teaser</itemRole> <itemClass>text</itemClass> <title/> <deck/> </itemMeta> <itemContent> </itemContent> </newsItem> </itemSet></root>
Questionnaire Body

161245_scarlet_fever_web.jpg
A 9-year old White female presented with 1 day of fever of 103° F, fatigue, and sore throat. She developed a papular, erythematous rash on the trunk that had a "sandpaper feel." The rash was not itchy.

Disallow All Ads
Content Gating
No Gating (article Unlocked/Free)
Alternative CME
Disqus Comments
Default
Use ProPublica
Hide sidebar & use full width
render the right sidebar.
Conference Recap Checkbox
Not Conference Recap
Clinical Edge
Display the Slideshow in this Article
Medscape Article
Display survey writer
Reuters content
Disable Inline Native ads
WebMD Article

A 95-year-old White male with hypertension presented with itchy patches and bullae on the trunk and extremities

Article Type
Changed
Thu, 11/03/2022 - 15:41

Bullous pemphigoid (BP) is the most common autoimmune bullous disease. It most frequently occurs in elderly patients and is associated with various predisposing factors, including HLA genes, comorbidities, aging, and trigger factors such as drugs, trauma, radiation, chemotherapy, and infections. The autoimmune reaction is mediated by a dysregulation of T cells in which IgG and IgE autoantibodies form against hemidesmosomal proteins (BP180 and BP230). These autoantibodies induce neutrophil activation, recruitment, and degradation in the basement membrane of the skin.

Typically, patients present with intense pruritus followed by an urticarial or eczematous eruption. Tense blisters and bullae occur commonly on the trunk and extremities. Drug-associated bullous pemphigoid (DABP) is a common manifestation of the disease with histologic and immunologic features similar to those of the idiopathic version. Eruptions can be triggered by systemic or topical medications, and incidence of these reactions may be related to a genetic predisposition for the disease.

Some research suggests that drug-induced changes to the antigenic properties of the epidermal basement membrane result in an augmented immune response, while others point to structural modification in these zones that stimulate the immune system. Thiol- and phenol-based drugs have been largely implicated in the development of DABP because they are capable of structural modification and disruption of the dermo-epidermal junction in the basement membrane.

DABP often presents with patients taking multiple medications. Some of the most common medications are gliptins, PD-1 inhibitors, diuretics, antibiotics, anti-inflammatory drugs, and ACE-inhibitors, and other cardiovascular drugs. DABP may present with mucosal eruptions unlike its idiopathic counterpart that is mostly contained to the skin.

BiluMartin_Donna_FLORIDA_web.jpg
Dr. Donna Bilu Martin

On this patient, two punch biopsies were taken. Histopathology revealed an eosinophil-rich subepidermal blister with a smooth epidermal undersurface consistent with bullous pemphigoid. Direct immunofluorescence was positive with a deposition of IgG and C3 at the epidermal side of salt split basement membrane zone.

Treatment for BP includes high potency topical and systemic steroids. Tetracyclines and niacinamide have been reported to improve the condition. Treatment is tailored to allow for cutaneous healing and control pruritus, but the physician must be mindful of the patient’s comorbidities and capacity for self-care. Prognosis is often better for DABP as withdrawal of the medication greatly accelerates clearance of the lesions. Worse prognosis is related to increased number of comorbidities and older age. Our patient’s BP is controlled currently with topical steroids and oral doxycycline.

This case and photo were submitted by Lucas Shapiro, BS, Nova Southeastern University College of Osteopathic Medicine, Tampa, and Dr. Bilu Martin.
 

Dr. Bilu Martin is a board-certified dermatologist in private practice at Premier Dermatology, MD, in Aventura, Fla. More diagnostic cases are available at mdedge.com/dermatology. To submit a case for possible publication, send an email to dermnews@mdedge.com.

References

1. Miyamoto D et al. An Bras Dermatol. 2019 Mar-Apr;94(2):133-46.

2. Moro et al. Biomolecules. 2020 Oct 10;10(10):1432.

3. Verheyden M et al. Acta Derm Venereol. 2020 Aug 17;100(15):adv00224.

Publications
Topics
Sections

Bullous pemphigoid (BP) is the most common autoimmune bullous disease. It most frequently occurs in elderly patients and is associated with various predisposing factors, including HLA genes, comorbidities, aging, and trigger factors such as drugs, trauma, radiation, chemotherapy, and infections. The autoimmune reaction is mediated by a dysregulation of T cells in which IgG and IgE autoantibodies form against hemidesmosomal proteins (BP180 and BP230). These autoantibodies induce neutrophil activation, recruitment, and degradation in the basement membrane of the skin.

Typically, patients present with intense pruritus followed by an urticarial or eczematous eruption. Tense blisters and bullae occur commonly on the trunk and extremities. Drug-associated bullous pemphigoid (DABP) is a common manifestation of the disease with histologic and immunologic features similar to those of the idiopathic version. Eruptions can be triggered by systemic or topical medications, and incidence of these reactions may be related to a genetic predisposition for the disease.

Some research suggests that drug-induced changes to the antigenic properties of the epidermal basement membrane result in an augmented immune response, while others point to structural modification in these zones that stimulate the immune system. Thiol- and phenol-based drugs have been largely implicated in the development of DABP because they are capable of structural modification and disruption of the dermo-epidermal junction in the basement membrane.

DABP often presents with patients taking multiple medications. Some of the most common medications are gliptins, PD-1 inhibitors, diuretics, antibiotics, anti-inflammatory drugs, and ACE-inhibitors, and other cardiovascular drugs. DABP may present with mucosal eruptions unlike its idiopathic counterpart that is mostly contained to the skin.

BiluMartin_Donna_FLORIDA_web.jpg
Dr. Donna Bilu Martin

On this patient, two punch biopsies were taken. Histopathology revealed an eosinophil-rich subepidermal blister with a smooth epidermal undersurface consistent with bullous pemphigoid. Direct immunofluorescence was positive with a deposition of IgG and C3 at the epidermal side of salt split basement membrane zone.

Treatment for BP includes high potency topical and systemic steroids. Tetracyclines and niacinamide have been reported to improve the condition. Treatment is tailored to allow for cutaneous healing and control pruritus, but the physician must be mindful of the patient’s comorbidities and capacity for self-care. Prognosis is often better for DABP as withdrawal of the medication greatly accelerates clearance of the lesions. Worse prognosis is related to increased number of comorbidities and older age. Our patient’s BP is controlled currently with topical steroids and oral doxycycline.

This case and photo were submitted by Lucas Shapiro, BS, Nova Southeastern University College of Osteopathic Medicine, Tampa, and Dr. Bilu Martin.
 

Dr. Bilu Martin is a board-certified dermatologist in private practice at Premier Dermatology, MD, in Aventura, Fla. More diagnostic cases are available at mdedge.com/dermatology. To submit a case for possible publication, send an email to dermnews@mdedge.com.

References

1. Miyamoto D et al. An Bras Dermatol. 2019 Mar-Apr;94(2):133-46.

2. Moro et al. Biomolecules. 2020 Oct 10;10(10):1432.

3. Verheyden M et al. Acta Derm Venereol. 2020 Aug 17;100(15):adv00224.

Bullous pemphigoid (BP) is the most common autoimmune bullous disease. It most frequently occurs in elderly patients and is associated with various predisposing factors, including HLA genes, comorbidities, aging, and trigger factors such as drugs, trauma, radiation, chemotherapy, and infections. The autoimmune reaction is mediated by a dysregulation of T cells in which IgG and IgE autoantibodies form against hemidesmosomal proteins (BP180 and BP230). These autoantibodies induce neutrophil activation, recruitment, and degradation in the basement membrane of the skin.

Typically, patients present with intense pruritus followed by an urticarial or eczematous eruption. Tense blisters and bullae occur commonly on the trunk and extremities. Drug-associated bullous pemphigoid (DABP) is a common manifestation of the disease with histologic and immunologic features similar to those of the idiopathic version. Eruptions can be triggered by systemic or topical medications, and incidence of these reactions may be related to a genetic predisposition for the disease.

Some research suggests that drug-induced changes to the antigenic properties of the epidermal basement membrane result in an augmented immune response, while others point to structural modification in these zones that stimulate the immune system. Thiol- and phenol-based drugs have been largely implicated in the development of DABP because they are capable of structural modification and disruption of the dermo-epidermal junction in the basement membrane.

DABP often presents with patients taking multiple medications. Some of the most common medications are gliptins, PD-1 inhibitors, diuretics, antibiotics, anti-inflammatory drugs, and ACE-inhibitors, and other cardiovascular drugs. DABP may present with mucosal eruptions unlike its idiopathic counterpart that is mostly contained to the skin.

Dr. Donna Bilu Martin

On this patient, two punch biopsies were taken. Histopathology revealed an eosinophil-rich subepidermal blister with a smooth epidermal undersurface consistent with bullous pemphigoid. Direct immunofluorescence was positive with a deposition of IgG and C3 at the epidermal side of salt split basement membrane zone.

Treatment for BP includes high potency topical and systemic steroids. Tetracyclines and niacinamide have been reported to improve the condition. Treatment is tailored to allow for cutaneous healing and control pruritus, but the physician must be mindful of the patient’s comorbidities and capacity for self-care. Prognosis is often better for DABP as withdrawal of the medication greatly accelerates clearance of the lesions. Worse prognosis is related to increased number of comorbidities and older age. Our patient’s BP is controlled currently with topical steroids and oral doxycycline.

This case and photo were submitted by Lucas Shapiro, BS, Nova Southeastern University College of Osteopathic Medicine, Tampa, and Dr. Bilu Martin.
 

Dr. Bilu Martin is a board-certified dermatologist in private practice at Premier Dermatology, MD, in Aventura, Fla. More diagnostic cases are available at mdedge.com/dermatology. To submit a case for possible publication, send an email to dermnews@mdedge.com.

References

1. Miyamoto D et al. An Bras Dermatol. 2019 Mar-Apr;94(2):133-46.

2. Moro et al. Biomolecules. 2020 Oct 10;10(10):1432.

3. Verheyden M et al. Acta Derm Venereol. 2020 Aug 17;100(15):adv00224.

Publications
Publications
Topics
Article Type
Sections
Teambase XML
<?xml version="1.0" encoding="UTF-8"?>
<!--$RCSfile: InCopy_agile.xsl,v $ $Revision: 1.35 $-->
<!--$RCSfile: drupal.xsl,v $ $Revision: 1.7 $-->
<root generator="drupal.xsl" gversion="1.7"> <header> <fileName>160811</fileName> <TBEID>0C0464ED.SIG</TBEID> <TBUniqueIdentifier>MD_0C0464ED</TBUniqueIdentifier> <newsOrJournal>News</newsOrJournal> <publisherName>Frontline Medical Communications</publisherName> <storyname>Dec. Make the Diagnosis</storyname> <articleType>353</articleType> <TBLocation>QC Done-All Pubs</TBLocation> <QCDate>20221103T092429</QCDate> <firstPublished>20221103T153514</firstPublished> <LastPublished>20221103T153514</LastPublished> <pubStatus qcode="stat:"/> <embargoDate/> <killDate/> <CMSDate>20221103T153514</CMSDate> <articleSource/> <facebookInfo/> <meetingNumber/> <byline>Shapiro and Bilu Martin</byline> <bylineText>LUCAS SHAPIRO AND DONNA BILU MARTIN, MD</bylineText> <bylineFull>LUCAS SHAPIRO AND DONNA BILU MARTIN, MD</bylineFull> <bylineTitleText/> <USOrGlobal/> <wireDocType/> <newsDocType>Column</newsDocType> <journalDocType/> <linkLabel/> <pageRange/> <citation/> <quizID/> <indexIssueDate/> <itemClass qcode="ninat:text"/> <provider qcode="provider:imng"> <name>IMNG Medical Media</name> <rightsInfo> <copyrightHolder> <name>Frontline Medical News</name> </copyrightHolder> <copyrightNotice>Copyright (c) 2015 Frontline Medical News, a Frontline Medical Communications Inc. company. All rights reserved. This material may not be published, broadcast, copied, or otherwise reproduced or distributed without the prior written permission of Frontline Medical Communications Inc.</copyrightNotice> </rightsInfo> </provider> <abstract/> <metaDescription>Bullous pemphigoid (BP) is the most common autoimmune bullous disease. It most frequently occurs in elderly patients</metaDescription> <articlePDF/> <teaserImage>290766</teaserImage> <teaser>Drug-associated bullous pemphigoid is a common manifestation of the disease with similar histologic and immunologic features to the idiopathic version.</teaser> <title>Bullous pemphigoid</title> <deck/> <disclaimer/> <AuthorList/> <articleURL/> <doi/> <pubMedID/> <publishXMLStatus/> <publishXMLVersion>1</publishXMLVersion> <useEISSN>0</useEISSN> <urgency/> <pubPubdateYear/> <pubPubdateMonth/> <pubPubdateDay/> <pubVolume/> <pubNumber/> <wireChannels/> <primaryCMSID/> <CMSIDs/> <keywords/> <seeAlsos/> <publications_g> <publicationData> <publicationCode>skin</publicationCode> <pubIssueName/> <pubArticleType/> <pubTopics/> <pubCategories/> <pubSections/> </publicationData> <publicationData> <publicationCode>card</publicationCode> <pubIssueName/> <pubArticleType/> <pubTopics/> <pubCategories/> <pubSections/> </publicationData> <publicationData> <publicationCode>fp</publicationCode> <pubIssueName/> <pubArticleType/> <pubTopics/> <pubCategories/> <pubSections/> </publicationData> <publicationData> <publicationCode>im</publicationCode> <pubIssueName/> <pubArticleType/> <pubTopics/> <pubCategories/> <pubSections/> </publicationData> </publications_g> <publications> <term canonical="true">13</term> <term>5</term> <term>15</term> <term>21</term> </publications> <sections> <term>52</term> <term canonical="true">87</term> </sections> <topics> <term canonical="true">39212</term> <term>229</term> <term>194</term> <term>203</term> </topics> <links> <link> <itemClass qcode="ninat:picture"/> <altRep contenttype="image/jpeg">images/240114ef.jpg</altRep> <description role="drol:caption"/> <description role="drol:credit">Courtesy Lucas Shapiro and Dr. Donna Bilu Martin</description> </link> <link> <itemClass qcode="ninat:picture"/> <altRep contenttype="image/jpeg">images/2400f1a6.jpg</altRep> <description role="drol:caption">Dr. Donna Bilu Martin</description> <description role="drol:credit"/> </link> </links> </header> <itemSet> <newsItem> <itemMeta> <itemRole>Main</itemRole> <itemClass>text</itemClass> <title>Bullous pemphigoid</title> <deck/> </itemMeta> <itemContent> <p><span class="tag metaDescription">Bullous pemphigoid (BP) is the most common autoimmune bullous disease. It most frequently occurs in elderly patients</span> and is associated with various predisposing factors, including HLA genes, comorbidities, aging, and trigger factors such as drugs, trauma, radiation, chemotherapy, and infections. The autoimmune reaction is mediated by a dysregulation of T cells in which IgG and IgE autoantibodies form against hemidesmosomal proteins (BP180 and BP230). These autoantibodies induce neutrophil activation, recruitment, and degradation in the basement membrane of the skin. </p> <p>Typically, patients present with intense pruritus followed by an urticarial or eczematous eruption. Tense blisters and bullae occur commonly on the trunk and extremities. Drug-associated bullous pemphigoid (DABP) is a common manifestation of the disease with histologic and immunologic features similar to those of the idiopathic version. Eruptions can be triggered by systemic or topical medications, and incidence of these reactions may be related to a genetic predisposition for the disease. <br/><br/>[[{"fid":"290766","view_mode":"medstat_image_flush_left","fields":{"format":"medstat_image_flush_left","field_file_image_alt_text[und][0][value]":"","field_file_image_credit[und][0][value]":"Courtesy Lucas Shapiro and Dr. Donna Bilu Martin","field_file_image_caption[und][0][value]":""},"type":"media","attributes":{"class":"media-element file-medstat_image_flush_left"}}]]Some research suggests that drug-induced changes to the antigenic properties of the epidermal basement membrane result in an augmented immune response, while others point to structural modification in these zones that stimulate the immune system. Thiol- and phenol-based drugs have been largely implicated in the development of DABP because they are capable of structural modification and disruption of the dermo-epidermal junction in the basement membrane.<br/><br/>DABP often presents with patients taking multiple medications. Some of the most common medications are gliptins, <span class="Hyperlink"><a href="https://reference.medscape.com/drugs/pd-1-pd-l1-inhibitors">PD-1 inhibitors</a></span>, diuretics, antibiotics, anti-inflammatory drugs, and ACE-inhibitors, and other cardiovascular drugs. DABP may present with mucosal eruptions unlike its idiopathic counterpart that is mostly contained to the skin. <br/><br/>[[{"fid":"271802","view_mode":"medstat_image_flush_right","fields":{"format":"medstat_image_flush_right","field_file_image_alt_text[und][0][value]":"Dr. Donna Bilu Martin, Premier Dermatology, MD, Aventura, Fla.","field_file_image_credit[und][0][value]":"","field_file_image_caption[und][0][value]":"Dr. Donna Bilu Martin"},"type":"media","attributes":{"class":"media-element file-medstat_image_flush_right"}}]]On this patient, two punch biopsies were taken. Histopathology revealed an eosinophil-rich subepidermal blister with a smooth epidermal undersurface consistent with bullous pemphigoid. Direct immunofluorescence was positive with a deposition of IgG and C3 at the epidermal side of salt split basement membrane zone.<br/><br/>Treatment for BP includes high potency topical and systemic steroids. Tetracyclines and niacinamide have been reported to improve the condition. Treatment is tailored to allow for cutaneous healing and control pruritus, but the physician must be mindful of the patient’s comorbidities and capacity for self-care. Prognosis is often better for DABP as withdrawal of the medication greatly accelerates clearance of the lesions. Worse prognosis is related to increased number of comorbidities and older age. Our patient’s BP is controlled currently with topical steroids and oral doxycycline.<br/><br/>This case and photo were submitted by Lucas Shapiro, BS, Nova Southeastern University College of Osteopathic Medicine, Tampa, and Dr. Bilu Martin.<br/><br/><br/><br/></p> <p> <em>Dr. Bilu Martin is a board-certified dermatologist in private practice at Premier Dermatology, MD, in Aventura, Fla. More diagnostic cases are available at <span class="Hyperlink"><a href="http://mdedge.com/dermatology">mdedge.com/dermatology</a></span>. To submit a case for possible publication, send an email to <span class="Hyperlink"><a href="mailto:dermnews%40mdedge.com?subject=">dermnews@mdedge.com</a></span>.</em> </p> <h2>References</h2> <p>1. Miyamoto D et al. <span class="Hyperlink"><a href="https://www.scielo.br/j/abd/a/BsschZpGNWjSKJ3rFKBCXjm/?lang=en">An Bras Dermatol. 2019 Mar-Apr;94(2):133-46</a></span>. <br/><br/>2. Moro et al. <span class="Hyperlink"><a href="https://www.mdpi.com/2218-273X/10/10/1432">Biomolecules. 2020 Oct 10;10(10):1432</a></span>.<br/><br/>3. Verheyden M et al. <span class="Hyperlink"><a href="https://medicaljournalssweden.se/actadv/article/view/1762">Acta Derm Venereol. 2020 Aug 17;100(15):adv00224</a></span>.</p> </itemContent> </newsItem> <newsItem> <itemMeta> <itemRole>teaser</itemRole> <itemClass>text</itemClass> <title/> <deck/> </itemMeta> <itemContent> </itemContent> </newsItem> </itemSet></root>
Questionnaire Body

A 95-year-old White male with hypertension presented with a history of very itchy patches and bullae on the trunk and extremities.

160811_bullous_pemphigoid_web.jpg
Lesions have come and gone over the past year. The patient takes many medications, including lisinopril for his hypertension.

Disallow All Ads
Content Gating
No Gating (article Unlocked/Free)
Alternative CME
Disqus Comments
Default
Use ProPublica
Hide sidebar & use full width
render the right sidebar.
Conference Recap Checkbox
Not Conference Recap
Clinical Edge
Display the Slideshow in this Article
Medscape Article
Display survey writer
Reuters content
Disable Inline Native ads
WebMD Article

A White female presented with pustules and erythematous macules on the left palm

Article Type
Changed
Tue, 02/07/2023 - 16:38

Psoriasis is an immune-mediated chronic inflammatory disease characterized by well-demarcated, scaly, erythematous plaques. Those who present with the condition often have a family history, which supports recent research uncovering various genes implicated in its pathogenesis. The disease is also associated with other systemic complications, most notably cardiovascular disease.

Palmoplantar psoriasis is a unique manifestation of psoriasis appearing in an acral distribution, but can coexist with plaque psoriasis, which is commonly found on extensor surfaces. This condition is found in a small percentage of patients with psoriasis and presentation varies from hyperkeratotic plaques to pustular lesions. The pustular form is known as palmoplantar pustulosis and is within the spectrum of palmoplantar psoriasis.

Psoriasis is typically a clinical diagnosis and its severity can be measured using the Psoriasis Area and Severity Index. If biopsy is performed, the histology demonstrates parakeratosis, orthokeratosis, loss of the stratum granulosum, and dilated vasculature with an inflammatory cell infiltrate. The keratinocytes present with abnormal differentiation and hyperplasia, and the presence of foci of neutrophils known as “Munro’s microabscesses” in the stratum corneum serve as the hallmark of histological diagnosis. However, it is important to note that appearance can vary based on the stage of the lesion and the subtype of psoriasis present.

Palmoplantar psoriasis can be especially limiting and difficult to treat because of its distribution. Topical steroids, topical vitamin D analogues, and narrow band ultraviolet light therapy can be effective for less severe cases. Methotrexate, biologic treatments, and apremilast can be used for more extensive disease.

Dr. Donna Bilu Martin

This patient is HLA-B27 positive and has uveitis. The presence of the HLA-B27 allele has been associated with inflammatory bowel disease, uveitis, psoriatic arthritis, and reactive arthritis. It has also been reported to be associated with pustular psoriasis. She responded well to topical steroids and vitamin D analogues.

This case and photo were submitted by Mr. Shapiro at Nova Southeastern University College of Osteopathic Medicine, Davie, Fla., and Dr. Bilu Martin.

Dr. Bilu Martin is a board-certified dermatologist in private practice at Premier Dermatology, MD, in Aventura, Fla. More diagnostic cases are available at mdedge.com/dermatology. To submit a case for possible publication, send an email to dermnews@mdedge.com.

References

1. Psoriasis: Overview and Diagnosis, in “Evidence-Based Psoriasis. Updates in Clinical Dermatology.” (Cham, Switzerland: Springer International, 2018).

2. Merola JF et al. Dermatol Ther. 2018 May;31(3):e12589.

3. Chung J et al. J Am Acad Dermatol. 2014 Oct;71(4):623-32.

Publications
Topics
Sections

Psoriasis is an immune-mediated chronic inflammatory disease characterized by well-demarcated, scaly, erythematous plaques. Those who present with the condition often have a family history, which supports recent research uncovering various genes implicated in its pathogenesis. The disease is also associated with other systemic complications, most notably cardiovascular disease.

Palmoplantar psoriasis is a unique manifestation of psoriasis appearing in an acral distribution, but can coexist with plaque psoriasis, which is commonly found on extensor surfaces. This condition is found in a small percentage of patients with psoriasis and presentation varies from hyperkeratotic plaques to pustular lesions. The pustular form is known as palmoplantar pustulosis and is within the spectrum of palmoplantar psoriasis.

Psoriasis is typically a clinical diagnosis and its severity can be measured using the Psoriasis Area and Severity Index. If biopsy is performed, the histology demonstrates parakeratosis, orthokeratosis, loss of the stratum granulosum, and dilated vasculature with an inflammatory cell infiltrate. The keratinocytes present with abnormal differentiation and hyperplasia, and the presence of foci of neutrophils known as “Munro’s microabscesses” in the stratum corneum serve as the hallmark of histological diagnosis. However, it is important to note that appearance can vary based on the stage of the lesion and the subtype of psoriasis present.

Palmoplantar psoriasis can be especially limiting and difficult to treat because of its distribution. Topical steroids, topical vitamin D analogues, and narrow band ultraviolet light therapy can be effective for less severe cases. Methotrexate, biologic treatments, and apremilast can be used for more extensive disease.

Dr. Donna Bilu Martin

This patient is HLA-B27 positive and has uveitis. The presence of the HLA-B27 allele has been associated with inflammatory bowel disease, uveitis, psoriatic arthritis, and reactive arthritis. It has also been reported to be associated with pustular psoriasis. She responded well to topical steroids and vitamin D analogues.

This case and photo were submitted by Mr. Shapiro at Nova Southeastern University College of Osteopathic Medicine, Davie, Fla., and Dr. Bilu Martin.

Dr. Bilu Martin is a board-certified dermatologist in private practice at Premier Dermatology, MD, in Aventura, Fla. More diagnostic cases are available at mdedge.com/dermatology. To submit a case for possible publication, send an email to dermnews@mdedge.com.

References

1. Psoriasis: Overview and Diagnosis, in “Evidence-Based Psoriasis. Updates in Clinical Dermatology.” (Cham, Switzerland: Springer International, 2018).

2. Merola JF et al. Dermatol Ther. 2018 May;31(3):e12589.

3. Chung J et al. J Am Acad Dermatol. 2014 Oct;71(4):623-32.

Psoriasis is an immune-mediated chronic inflammatory disease characterized by well-demarcated, scaly, erythematous plaques. Those who present with the condition often have a family history, which supports recent research uncovering various genes implicated in its pathogenesis. The disease is also associated with other systemic complications, most notably cardiovascular disease.

Palmoplantar psoriasis is a unique manifestation of psoriasis appearing in an acral distribution, but can coexist with plaque psoriasis, which is commonly found on extensor surfaces. This condition is found in a small percentage of patients with psoriasis and presentation varies from hyperkeratotic plaques to pustular lesions. The pustular form is known as palmoplantar pustulosis and is within the spectrum of palmoplantar psoriasis.

Psoriasis is typically a clinical diagnosis and its severity can be measured using the Psoriasis Area and Severity Index. If biopsy is performed, the histology demonstrates parakeratosis, orthokeratosis, loss of the stratum granulosum, and dilated vasculature with an inflammatory cell infiltrate. The keratinocytes present with abnormal differentiation and hyperplasia, and the presence of foci of neutrophils known as “Munro’s microabscesses” in the stratum corneum serve as the hallmark of histological diagnosis. However, it is important to note that appearance can vary based on the stage of the lesion and the subtype of psoriasis present.

Palmoplantar psoriasis can be especially limiting and difficult to treat because of its distribution. Topical steroids, topical vitamin D analogues, and narrow band ultraviolet light therapy can be effective for less severe cases. Methotrexate, biologic treatments, and apremilast can be used for more extensive disease.

Dr. Donna Bilu Martin

This patient is HLA-B27 positive and has uveitis. The presence of the HLA-B27 allele has been associated with inflammatory bowel disease, uveitis, psoriatic arthritis, and reactive arthritis. It has also been reported to be associated with pustular psoriasis. She responded well to topical steroids and vitamin D analogues.

This case and photo were submitted by Mr. Shapiro at Nova Southeastern University College of Osteopathic Medicine, Davie, Fla., and Dr. Bilu Martin.

Dr. Bilu Martin is a board-certified dermatologist in private practice at Premier Dermatology, MD, in Aventura, Fla. More diagnostic cases are available at mdedge.com/dermatology. To submit a case for possible publication, send an email to dermnews@mdedge.com.

References

1. Psoriasis: Overview and Diagnosis, in “Evidence-Based Psoriasis. Updates in Clinical Dermatology.” (Cham, Switzerland: Springer International, 2018).

2. Merola JF et al. Dermatol Ther. 2018 May;31(3):e12589.

3. Chung J et al. J Am Acad Dermatol. 2014 Oct;71(4):623-32.

Publications
Publications
Topics
Article Type
Sections
Teambase XML
<?xml version="1.0" encoding="UTF-8"?>
<!--$RCSfile: InCopy_agile.xsl,v $ $Revision: 1.35 $-->
<!--$RCSfile: drupal.xsl,v $ $Revision: 1.7 $-->
<root generator="drupal.xsl" gversion="1.7"> <header> <fileName>160256</fileName> <TBEID>0C04596C.SIG</TBEID> <TBUniqueIdentifier>MD_0C04596C</TBUniqueIdentifier> <newsOrJournal>News</newsOrJournal> <publisherName>Frontline Medical Communications</publisherName> <storyname>November Make the Dx</storyname> <articleType>353</articleType> <TBLocation>QC Done-All Pubs</TBLocation> <QCDate>20221011T083316</QCDate> <firstPublished>20221013T164314</firstPublished> <LastPublished>20221013T164314</LastPublished> <pubStatus qcode="stat:"/> <embargoDate>20221014T090000</embargoDate> <killDate/> <CMSDate>20221014T090000</CMSDate> <articleSource/> <facebookInfo/> <meetingNumber/> <byline>Donna Bilu Martin</byline> <bylineText>LUCAS SHAPIRO, BS, AND DONNA BILU MARTIN, MD</bylineText> <bylineFull>LUCAS SHAPIRO, BS, AND DONNA BILU MARTIN, MD</bylineFull> <bylineTitleText/> <USOrGlobal/> <wireDocType/> <newsDocType/> <journalDocType/> <linkLabel/> <pageRange/> <citation/> <quizID/> <indexIssueDate/> <itemClass qcode="ninat:text"/> <provider qcode="provider:imng"> <name>IMNG Medical Media</name> <rightsInfo> <copyrightHolder> <name>Frontline Medical News</name> </copyrightHolder> <copyrightNotice>Copyright (c) 2015 Frontline Medical News, a Frontline Medical Communications Inc. company. All rights reserved. This material may not be published, broadcast, copied, or otherwise reproduced or distributed without the prior written permission of Frontline Medical Communications Inc.</copyrightNotice> </rightsInfo> </provider> <abstract/> <metaDescription>Palmoplantar psoriasis is a unique manifestation of psoriasis appearing in an acral distribution, but can coexist with plaque psoriasis, which is commonly found</metaDescription> <articlePDF/> <teaserImage>290019</teaserImage> <teaser>This condition is found in a small percentage of patients with psoriasis and presentation varies from hyperkeratotic plaques to pustular lesions.</teaser> <title>Palmoplantar psoriasis</title> <deck/> <disclaimer/> <AuthorList/> <articleURL/> <doi/> <pubMedID/> <publishXMLStatus/> <publishXMLVersion>1</publishXMLVersion> <useEISSN>0</useEISSN> <urgency/> <pubPubdateYear/> <pubPubdateMonth/> <pubPubdateDay/> <pubVolume/> <pubNumber/> <wireChannels/> <primaryCMSID/> <CMSIDs/> <keywords/> <seeAlsos/> <publications_g> <publicationData> <publicationCode>skin</publicationCode> <pubIssueName/> <pubArticleType/> <pubTopics/> <pubCategories/> <pubSections/> </publicationData> <publicationData> <publicationCode>fp</publicationCode> <pubIssueName/> <pubArticleType/> <pubTopics/> <pubCategories/> <pubSections/> </publicationData> <publicationData> <publicationCode>im</publicationCode> <pubIssueName/> <pubArticleType/> <pubTopics/> <pubCategories/> <pubSections/> </publicationData> <publicationData> <publicationCode>rn</publicationCode> <pubIssueName/> <pubArticleType/> <pubTopics/> <pubCategories/> <pubSections/> </publicationData> </publications_g> <publications> <term canonical="true">13</term> <term>15</term> <term>21</term> <term>26</term> </publications> <sections> <term>52</term> <term canonical="true">87</term> </sections> <topics> <term canonical="true">281</term> <term>282</term> <term>203</term> </topics> <links> <link> <itemClass qcode="ninat:picture"/> <altRep contenttype="image/jpeg">images/240113b9.jpg</altRep> <description role="drol:caption"/> <description role="drol:credit"/> </link> <link> <itemClass qcode="ninat:picture"/> <altRep contenttype="image/jpeg">images/2400f1a6.jpg</altRep> <description role="drol:caption">Dr. Donna Bilu Martin</description> <description role="drol:credit"/> </link> </links> </header> <itemSet> <newsItem> <itemMeta> <itemRole>Main</itemRole> <itemClass>text</itemClass> <title>Palmoplantar psoriasis</title> <deck/> </itemMeta> <itemContent> <p>Psoriasis is an immune-mediated chronic inflammatory disease characterized by well-demarcated, scaly, erythematous plaques. Those who present with the condition often have a family history, which supports recent research uncovering various genes implicated in its pathogenesis. The disease is also associated with other systemic complications, most notably cardiovascular disease.</p> <p><span class="tag metaDescription">Palmoplantar psoriasis is a unique manifestation of psoriasis appearing in an acral distribution, but can coexist with plaque psoriasis, which is commonly found on extensor surfaces.</span> This condition is found in a small percentage of patients with psoriasis and presentation varies from hyperkeratotic plaques to pustular lesions. The pustular form is known as palmoplantar pustulosis and is within the spectrum of palmoplantar psoriasis. <br/><br/>[[{"fid":"290019","view_mode":"medstat_image_flush_left","fields":{"format":"medstat_image_flush_left","field_file_image_alt_text[und][0][value]":"","field_file_image_credit[und][0][value]":"","field_file_image_caption[und][0][value]":""},"type":"media","attributes":{"class":"media-element file-medstat_image_flush_left"}}]]Psoriasis is typically a clinical diagnosis and its severity can be measured using the Psoriasis Area and Severity Index. If biopsy is performed, the histology demonstrates parakeratosis, orthokeratosis, loss of the stratum granulosum, and dilated vasculature with an inflammatory cell infiltrate. The keratinocytes present with abnormal differentiation and hyperplasia, and the presence of foci of neutrophils known as “Munro’s microabscesses” in the stratum corneum serve as the hallmark of histological diagnosis. However, it is important to note that appearance can vary based on the stage of the lesion and the subtype of psoriasis present. <br/><br/>Palmoplantar psoriasis can be especially limiting and difficult to treat because of its distribution. Topical steroids, topical vitamin D analogues, and narrow band ultraviolet light therapy can be effective for less severe cases. Methotrexate, biologic treatments, and apremilast can be used for more extensive disease. <br/><br/>[[{"fid":"271802","view_mode":"medstat_image_flush_right","fields":{"format":"medstat_image_flush_right","field_file_image_alt_text[und][0][value]":"Dr. Donna Bilu Martin, Premier Dermatology, MD, Aventura, Fla.","field_file_image_credit[und][0][value]":"","field_file_image_caption[und][0][value]":"Dr. Donna Bilu Martin"},"type":"media","attributes":{"class":"media-element file-medstat_image_flush_right"}}]]This patient is HLA-B27 positive and has uveitis. The presence of the HLA-B27 allele has been associated with inflammatory bowel disease, uveitis, psoriatic arthritis, and reactive arthritis. It has also been reported to be associated with pustular psoriasis. She responded well to topical steroids and vitamin D analogues.<br/><br/>This case and photo were submitted by Mr. Shapiro at Nova Southeastern University College of Osteopathic Medicine, Davie, Fla., and Dr. Bilu Martin. </p> <p> <em>Dr. Bilu Martin is a board-certified dermatologist in private practice at Premier Dermatology, MD, in Aventura, Fla. More diagnostic cases are available at mdedge.com/dermatology. To submit a case for possible publication, send an email to dermnews@mdedge.com.</em> </p> <h2>References</h2> <p>1. <span class="Hyperlink"><a href="https://link.springer.com/chapter/10.1007/978-3-319-90107-7_1">Psoriasis: Overview and Diagnosis</a>, </span>in “Evidence-Based Psoriasis. Updates in Clinical Dermatology.” (Cham, Switzerland: Springer International, 2018). <br/><br/>2. Merola JF et al. <span class="Hyperlink"><a href="https://onlinelibrary.wiley.com/doi/10.1111/dth.12589">Dermatol Ther. 2018 May;31(3):e12589</a></span>.<br/><br/>3. Chung J et al. <span class="Hyperlink"><a href="https://www.jaad.org/article/S0190-9622(14)01421-2/fulltext">J Am Acad Dermatol. 2014 Oct;71(4):623-32</a></span>.</p> </itemContent> </newsItem> <newsItem> <itemMeta> <itemRole>teaser</itemRole> <itemClass>text</itemClass> <title/> <deck/> </itemMeta> <itemContent> </itemContent> </newsItem> </itemSet></root>
Questionnaire Body

160256_palm_psoriasis_web.jpg
A 53-year-old White female presented with itchy pustules and erythematous macules on the left palm. She has a 20 plus-year history of similar lesions on the palms and soles that come and go, and a history of uveitis. Her sister has a history of Crohn's disease.

Disallow All Ads
Content Gating
No Gating (article Unlocked/Free)
Alternative CME
Disqus Comments
Default
Use ProPublica
Hide sidebar & use full width
render the right sidebar.
Conference Recap Checkbox
Not Conference Recap
Clinical Edge
Display the Slideshow in this Article
Medscape Article
Display survey writer
Reuters content
Disable Inline Native ads
WebMD Article

​​​​​​​A healthy White male presented with a rash consisting of erythematous to purpuric macules

Article Type
Changed
Mon, 07/18/2022 - 14:31

Vasculitis is a process in which blood vessels become inflamed and necrotic. Classic small vessel vasculitis reveals a leukocytoclastic vasculitis and most commonly presents as palpable purpura. Exercise-induced vasculitis (EIV) is a benign form of vasculitis involving the small vessels, brought on by exercise. It is also known as “golfer’s vasculitis.” A form of EIV has been described in the literature as “Disney dermatitis.” It is often seen in healthy adults after a long day of walking at the parks. Other forms of exercise, such as jogging, hiking, or swimming, may also cause the condition.

Clinically, EIV affects the lower legs and presents as purpuric macules. Edema may be present. Lesions may be asymptomatic or may present with pruritus or burning. Diagnosis is often made clinically. Skin biopsies for H&E and DIF (direct immunofluorescence) can help distinguish the type of vasculitis that is present. Laboratory tests may be needed to exclude other causes of vasculitis. Episodes may be recurrent.

Henoch-Schönlein purpura (HSP), also called anaphylactoid purpura, is a subtype of small-vessel vasculitis where IgA immunoglobulin is deposited in the vessel walls. It is the most common form of vasculitis is children (usually ages 4-8). In addition to skin, organs such as joints, kidneys, and intestines can be involved. Schamberg’s disease, or capillaritis, is also called pigmented purpura. In this benign condition, leakage from capillaries results in erythematous to brown patches on the lower extremities. A true vasculitis is not seen. The brown discoloration is due to hemosiderin deposition. Cryoglobulinemia is a rare condition in which abnormal immunoglobulin complexes deposit in tissues and vessels. Leukocytoclastic vasculitis is present in small vessels. Palpable purpura and livedo may be seen clinically, and systemic symptoms may be present.

Dr. Donna Bilu Martin

Treatment of EIV is largely supportive as lesions will resolve on their own over 3-4 weeks. Postinflammatory hyperpigmentation may result. Temporary cessation of exercise and compression stockings can help speed up the resolution of lesions. Systemic medications used in the treatment of severe vasculitis, such as systemic steroids, dapsone, and colchicine, are not needed in EIV.
 

Dr. Bilu Martin is a board-certified dermatologist in private practice at Premier Dermatology, MD, in Aventura, Fla. More diagnostic cases are available at mdedge.com/dermatology. To submit a case for possible publication, send an email to dermnews@mdedge.com.

Publications
Topics
Sections

Vasculitis is a process in which blood vessels become inflamed and necrotic. Classic small vessel vasculitis reveals a leukocytoclastic vasculitis and most commonly presents as palpable purpura. Exercise-induced vasculitis (EIV) is a benign form of vasculitis involving the small vessels, brought on by exercise. It is also known as “golfer’s vasculitis.” A form of EIV has been described in the literature as “Disney dermatitis.” It is often seen in healthy adults after a long day of walking at the parks. Other forms of exercise, such as jogging, hiking, or swimming, may also cause the condition.

Clinically, EIV affects the lower legs and presents as purpuric macules. Edema may be present. Lesions may be asymptomatic or may present with pruritus or burning. Diagnosis is often made clinically. Skin biopsies for H&E and DIF (direct immunofluorescence) can help distinguish the type of vasculitis that is present. Laboratory tests may be needed to exclude other causes of vasculitis. Episodes may be recurrent.

Henoch-Schönlein purpura (HSP), also called anaphylactoid purpura, is a subtype of small-vessel vasculitis where IgA immunoglobulin is deposited in the vessel walls. It is the most common form of vasculitis is children (usually ages 4-8). In addition to skin, organs such as joints, kidneys, and intestines can be involved. Schamberg’s disease, or capillaritis, is also called pigmented purpura. In this benign condition, leakage from capillaries results in erythematous to brown patches on the lower extremities. A true vasculitis is not seen. The brown discoloration is due to hemosiderin deposition. Cryoglobulinemia is a rare condition in which abnormal immunoglobulin complexes deposit in tissues and vessels. Leukocytoclastic vasculitis is present in small vessels. Palpable purpura and livedo may be seen clinically, and systemic symptoms may be present.

Dr. Donna Bilu Martin

Treatment of EIV is largely supportive as lesions will resolve on their own over 3-4 weeks. Postinflammatory hyperpigmentation may result. Temporary cessation of exercise and compression stockings can help speed up the resolution of lesions. Systemic medications used in the treatment of severe vasculitis, such as systemic steroids, dapsone, and colchicine, are not needed in EIV.
 

Dr. Bilu Martin is a board-certified dermatologist in private practice at Premier Dermatology, MD, in Aventura, Fla. More diagnostic cases are available at mdedge.com/dermatology. To submit a case for possible publication, send an email to dermnews@mdedge.com.

Vasculitis is a process in which blood vessels become inflamed and necrotic. Classic small vessel vasculitis reveals a leukocytoclastic vasculitis and most commonly presents as palpable purpura. Exercise-induced vasculitis (EIV) is a benign form of vasculitis involving the small vessels, brought on by exercise. It is also known as “golfer’s vasculitis.” A form of EIV has been described in the literature as “Disney dermatitis.” It is often seen in healthy adults after a long day of walking at the parks. Other forms of exercise, such as jogging, hiking, or swimming, may also cause the condition.

Clinically, EIV affects the lower legs and presents as purpuric macules. Edema may be present. Lesions may be asymptomatic or may present with pruritus or burning. Diagnosis is often made clinically. Skin biopsies for H&E and DIF (direct immunofluorescence) can help distinguish the type of vasculitis that is present. Laboratory tests may be needed to exclude other causes of vasculitis. Episodes may be recurrent.

Henoch-Schönlein purpura (HSP), also called anaphylactoid purpura, is a subtype of small-vessel vasculitis where IgA immunoglobulin is deposited in the vessel walls. It is the most common form of vasculitis is children (usually ages 4-8). In addition to skin, organs such as joints, kidneys, and intestines can be involved. Schamberg’s disease, or capillaritis, is also called pigmented purpura. In this benign condition, leakage from capillaries results in erythematous to brown patches on the lower extremities. A true vasculitis is not seen. The brown discoloration is due to hemosiderin deposition. Cryoglobulinemia is a rare condition in which abnormal immunoglobulin complexes deposit in tissues and vessels. Leukocytoclastic vasculitis is present in small vessels. Palpable purpura and livedo may be seen clinically, and systemic symptoms may be present.

Dr. Donna Bilu Martin

Treatment of EIV is largely supportive as lesions will resolve on their own over 3-4 weeks. Postinflammatory hyperpigmentation may result. Temporary cessation of exercise and compression stockings can help speed up the resolution of lesions. Systemic medications used in the treatment of severe vasculitis, such as systemic steroids, dapsone, and colchicine, are not needed in EIV.
 

Dr. Bilu Martin is a board-certified dermatologist in private practice at Premier Dermatology, MD, in Aventura, Fla. More diagnostic cases are available at mdedge.com/dermatology. To submit a case for possible publication, send an email to dermnews@mdedge.com.

Publications
Publications
Topics
Article Type
Sections
Teambase XML
<?xml version="1.0" encoding="UTF-8"?>
<!--$RCSfile: InCopy_agile.xsl,v $ $Revision: 1.35 $-->
<!--$RCSfile: drupal.xsl,v $ $Revision: 1.7 $-->
<root generator="drupal.xsl" gversion="1.7"> <header> <fileName>159050</fileName> <TBEID>0C043EC1.SIG</TBEID> <TBUniqueIdentifier>MD_0C043EC1</TBUniqueIdentifier> <newsOrJournal>News</newsOrJournal> <publisherName>Frontline Medical Communications</publisherName> <storyname>August Make the Dx</storyname> <articleType>353</articleType> <TBLocation>QC Done-All Pubs</TBLocation> <QCDate>20220718T124652</QCDate> <firstPublished>20220718T142428</firstPublished> <LastPublished>20220718T142428</LastPublished> <pubStatus qcode="stat:"/> <embargoDate/> <killDate/> <CMSDate>20220718T142428</CMSDate> <articleSource/> <facebookInfo/> <meetingNumber/> <byline>Donna Bilu Martin</byline> <bylineText>DONNA BILU MARTIN, MD</bylineText> <bylineFull>DONNA BILU MARTIN, MD</bylineFull> <bylineTitleText/> <USOrGlobal/> <wireDocType/> <newsDocType>Column</newsDocType> <journalDocType/> <linkLabel/> <pageRange/> <citation/> <quizID/> <indexIssueDate/> <itemClass qcode="ninat:text"/> <provider qcode="provider:imng"> <name>IMNG Medical Media</name> <rightsInfo> <copyrightHolder> <name>Frontline Medical News</name> </copyrightHolder> <copyrightNotice>Copyright (c) 2015 Frontline Medical News, a Frontline Medical Communications Inc. company. All rights reserved. This material may not be published, broadcast, copied, or otherwise reproduced or distributed without the prior written permission of Frontline Medical Communications Inc.</copyrightNotice> </rightsInfo> </provider> <abstract/> <metaDescription>Exercise-induced vasculitis (EIV) is a benign form of vasculitis involving the small vessels, brought on by exercise. It is also known as “golfer’s vasculitis</metaDescription> <articlePDF/> <teaserImage>288069</teaserImage> <title>Exercise-induced vasculitis</title> <deck/> <disclaimer/> <AuthorList/> <articleURL/> <doi/> <pubMedID/> <publishXMLStatus/> <publishXMLVersion>1</publishXMLVersion> <useEISSN>0</useEISSN> <urgency/> <pubPubdateYear/> <pubPubdateMonth/> <pubPubdateDay/> <pubVolume/> <pubNumber/> <wireChannels/> <primaryCMSID/> <CMSIDs/> <keywords/> <seeAlsos/> <publications_g> <publicationData> <publicationCode>skin</publicationCode> <pubIssueName/> <pubArticleType/> <pubTopics/> <pubCategories/> <pubSections/> </publicationData> <publicationData> <publicationCode>fp</publicationCode> <pubIssueName/> <pubArticleType/> <pubTopics/> <pubCategories/> <pubSections/> </publicationData> <publicationData> <publicationCode>im</publicationCode> <pubIssueName/> <pubArticleType/> <pubTopics/> <pubCategories/> <pubSections/> </publicationData> </publications_g> <publications> <term canonical="true">13</term> <term>15</term> <term>21</term> </publications> <sections> <term canonical="true">87</term> <term>52</term> </sections> <topics> <term canonical="true">39212</term> <term>203</term> </topics> <links> <link> <itemClass qcode="ninat:picture"/> <altRep contenttype="image/jpeg">images/24010f62.jpg</altRep> <description role="drol:caption"/> <description role="drol:credit">Courtesy Dr. Bilu Martin</description> </link> <link> <itemClass qcode="ninat:picture"/> <altRep contenttype="image/jpeg">images/2400f1a6.jpg</altRep> <description role="drol:caption">Dr. Donna Bilu Martin</description> <description role="drol:credit"/> </link> </links> </header> <itemSet> <newsItem> <itemMeta> <itemRole>Main</itemRole> <itemClass>text</itemClass> <title>Exercise-induced vasculitis</title> <deck/> </itemMeta> <itemContent> <p>Vasculitis is a process in which blood vessels become inflamed and necrotic. Classic small vessel vasculitis reveals a leukocytoclastic vasculitis and most commonly presents as palpable purpura. <span class="tag metaDescription">Exercise-induced vasculitis (EIV) is a benign form of vasculitis involving the small vessels, brought on by exercise. It is also known as “golfer’s vasculitis</span>.” A form of EIV has been described in the literature as “Disney dermatitis.” It is often seen in healthy adults after a long day of walking at the parks. Other forms of exercise, such as jogging, hiking, or swimming, may also cause the condition.</p> <p>Clinically, EIV affects the lower legs and presents as purpuric macules. Edema may be present. Lesions may be asymptomatic or may present with pruritus or burning. Diagnosis is often made clinically. Skin biopsies for H&amp;E and DIF (direct immunofluorescence) can help distinguish the type of vasculitis that is present. Laboratory tests may be needed to exclude other causes of vasculitis. Episodes may be recurrent. <br/><br/>[[{"fid":"288069","view_mode":"medstat_image_flush_left","fields":{"format":"medstat_image_flush_left","field_file_image_alt_text[und][0][value]":"","field_file_image_credit[und][0][value]":"Courtesy Dr. Bilu Martin","field_file_image_caption[und][0][value]":""},"type":"media","attributes":{"class":"media-element file-medstat_image_flush_left"}}]]Henoch-Schönlein purpura (HSP), also called anaphylactoid purpura, is a subtype of small-vessel vasculitis where IgA immunoglobulin is deposited in the vessel walls. It is the most common form of vasculitis is children (usually ages 4-8). In addition to skin, organs such as joints, kidneys, and intestines can be involved. Schamberg’s disease, or capillaritis, is also called pigmented purpura. In this benign condition, leakage from capillaries results in erythematous to brown patches on the lower extremities. A true vasculitis is not seen. The brown discoloration is due to hemosiderin deposition. Cryoglobulinemia is a rare condition in which abnormal immunoglobulin complexes deposit in tissues and vessels. Leukocytoclastic vasculitis is present in small vessels. Palpable purpura and livedo may be seen clinically, and systemic symptoms may be present. <br/><br/>[[{"fid":"271802","view_mode":"medstat_image_flush_right","fields":{"format":"medstat_image_flush_right","field_file_image_alt_text[und][0][value]":"Dr. Donna Bilu Martin, Premier Dermatology, MD, Aventura, Fla.","field_file_image_credit[und][0][value]":"","field_file_image_caption[und][0][value]":"Dr. Donna Bilu Martin"},"type":"media","attributes":{"class":"media-element file-medstat_image_flush_right"}}]]Treatment of EIV is largely supportive as lesions will resolve on their own over 3-4 weeks. Postinflammatory hyperpigmentation may result. Temporary cessation of exercise and compression stockings can help speed up the resolution of lesions. Systemic medications used in the treatment of severe vasculitis, such as systemic steroids, dapsone, and colchicine, are not needed in EIV. <br/><br/></p> <p> <em>Dr. Bilu Martin is a board-certified dermatologist in private practice at Premier Dermatology, MD, in Aventura, Fla. More diagnostic cases are available at <span class="Hyperlink"><a href="http://mdedge.com/dermatology">mdedge.com/dermatology</a></span>. To submit a case for possible publication, send an email to <span class="Hyperlink"><a href="mailto:dermnews%40mdedge.com?subject=">dermnews@mdedge.com</a></span>.</em> </p> </itemContent> </newsItem> <newsItem> <itemMeta> <itemRole>teaser</itemRole> <itemClass>text</itemClass> <title/> <deck/> </itemMeta> <itemContent> <p>A form of exercise-induced vasculitis has been described in the literature as “Disney dermatitis.”</p> </itemContent> </newsItem> </itemSet></root>
Questionnaire Body

159050_foot_web.jpg
A 28-year-old healthy White male presented with an asymptomatic rash consisting of erythematous to purpuric macules on his lower extremities. He had been in Disney World a few days prior to presentation and had been walking all over the parks for hours. He had no systemic symptoms.

Disallow All Ads
Content Gating
No Gating (article Unlocked/Free)
Alternative CME
Disqus Comments
Default
Use ProPublica
Hide sidebar & use full width
render the right sidebar.
Conference Recap Checkbox
Not Conference Recap
Clinical Edge
Display the Slideshow in this Article
Medscape Article
Display survey writer
Reuters content
Disable Inline Native ads
WebMD Article